You are on page 1of 70

Step 2 CK (CCSSA) Form 7 – Step Prep 30/10/21, 12:51 AM

STEP PREP
Helping medical students navigate 3rd year exams

MARCH 8, 2020 BY STEPPREP12345

Step 2 CK (CCSSA) Form 7


These answer explanations are and always will be free. However, given multiple email
requests, I will post my Venmo (@Adam-Zakaria-SLO) if you want to send a few
dollars to show your support for the website. I also offer reasonably priced tutoring,
so please reach out using the menu option listed above if you would like personalized
support. Good luck with your exams!

Exam section 1:

1: A previously healthy 9-year-old boy is brought to the physician…

Ankylosing spondylitis

Child who presents with inflammatory back pain (worse upon waking in morning)
with involvement of sacroiliac joint and decreased flexion at waist (2 features that
are highly associated/specific for ankylosing spondylitis) and elevated ESR (further
supporting inflammatory back pain) with imaging showing vertebral body squaring
and fusion of sacroiliac joint, most consistent with ankylosing spondylitis
Key idea: Back pain that is worse upon waking in morning = Inflammatory back pain =
PAIR diagnoses: Psoriatic arthritis, Ankylosing spondylitis, Inflammatory bowel
disease, Reiter/Reactive arthritis (at least on NBME exams)

https://step-prep.org/step-2-ck-ccssa-form-6-2/ Page 1 of 70
Step 2 CK (CCSSA) Form 7 – Step Prep 30/10/21, 12:51 AM

2. A 15-year-old girl is brought to the physician because of low back…

Compression fracture

Young girl with a history of chronic corticosteroid use and cushingoid appearance
who presents with constant back pain with vertebral point tenderness with
imaging showing vertebrae with increased lucency, most consistent with a
compression fracture in the setting of steroid-induced osteoporosis
Key idea: Vertebral compression fracture comes in 2 flavors (1) Chronic
fracture: Painless with progressive kyphosis and loss of stature (2) Acute
fracture: Low back pain with decreased spinal mobility + tenderness at affected level
Key idea: Causes of point tenderness over vertebral body includes compression
vertebral fractures, osteomyelitis and metastatic disease to vertebral bone
Risk factors for osteoporosis –> compression fracture: Low weight (osteoporosis),
increased age, women, chronic corticosteroid use or Cushing syndrome

3. Over the past month, a 72-year-old woman has had indigestion…

Exercise stress test

Patient with “indigestion” who has exertional chest pain that is unrelated to eating
or relieved by antacids, racing concern for stable angina that should be worked up
by exercise stress test

4. A previously healthy 62-year-old woman comes to the physician…

Barium swallow

Older woman with dysphagia to solids and liquids with intermittent regurgitation
of undigested food and halitosis, most consistent with achalasia which can be
worked up with barium swallow (or esophageal manometry)
Key idea: Dysphagia to solids AND LIQUIDS is due to problems with esophageal
motility, whereas dysphagia with solids that later leads to dysphagia to liquids
more associated with obstructing mass lesions (cancer, esophageal stricture, etc.)
that should be worked up with endoscopy

https://step-prep.org/step-2-ck-ccssa-form-6-2/ Page 2 of 70
Step 2 CK (CCSSA) Form 7 – Step Prep 30/10/21, 12:51 AM

5. A 14-month-old boy is brought for a well-child examination…

Reassurance

Key idea: Normal age for child to begin walking is between 12 and 15 months

6. A 67-year-old man comes to the physician because of a 1-month…

Randomized, controlled clinical trial

Key idea: Randomized, controlled clinical trial is the best type of research study for
getting accurate/useful results

7. A 32-year-old woman is brought to the emergency department…

Triglycerides

Young woman with signs of pancreatitis (epigastric abdominal pain with


nausea/vomiting) and elevated amylase, most consistent with acute pancreatitis
Diagnosis of acute pancreatitis requires 2/3: (1) Acute epigastric pain radiating to
the back (2) Increased serum amylase or lipase to at least 3X upper limit of normal
(3) Characteristic imaging findings
Key idea: Two most common causes of acute pancreatitis are gallstones and
alcohol, so in this patient who had a cholecystectomy and does not use alcohol, we
would think of other potential causes such as hypertriglyceridemia, hypercalcemia
(this patient has normal calcium), steroids, autoimmune pancreatitis, post-ERCP,
drugs (diuretics, IBD drugs, anticonvulsants, etc.)

8. An 18-year-old woman comes to the physician because of a…

Splenectomy

Young woman with a family history of anemia presents with jaundice, splenomegaly
and anemia with normal reticulocyte count, negative Coombs test and elevated
mean corpuscular hemoglobin concentration with a peripheral smear showing
spherocytes (RBCs without area of central pallor), most consistent with hereditary

https://step-prep.org/step-2-ck-ccssa-form-6-2/ Page 3 of 70
Step 2 CK (CCSSA) Form 7 – Step Prep 30/10/21, 12:51 AM

spherocytosis
Key idea: Hereditary spherocytosis is primarily transmitted in an autosomal
dominant fashion and is treated with splenectomy (because the hemolysis occurs in
the spleen because the splenic macrophages recognize the spherocytes as being
misshapen)

9. A 17-year-old girl comes to the physician because of a 4-month…

Pilosebaceous follicles

Adolescent with a polymorphic facial rash with comedones and inflammatory


papules, most consistent with acne which is due to pathology within pilosebaceous
follicles
Apocrine glands associated with hidradenitis suppurativa, epidermal-dermal
junction associated with bullous pemphigoid and panniculitis associated with
erythema nodosum

10. During examination prior to participation in school sports…

Repeat blood pressure measurement in 4 weeks

Key idea: In a patient presenting with hypertension (particularly a child), you always
should start with a trial of lifestyle interventions (playing sports in this patient’s
case) before thinking about using blood pressure medications

11. A previously healthy 24-year-old woman comes to the…

Decreased acethylcholine receptors

Presentation is most consistent with myasthenia gravis (young woman or older man
presenting with dysphagia, dysarthria and eye weakness that is worse at the end of
the day), which is caused by antibodies attacking acetylcholine receptors and
leading to reduced acetylcholine receptors
Decreased release of Ach from motor nerve terminals = Lambert-Eaton syndrome
Degeneration of muscle fibers = Muscular dystrophy

https://step-prep.org/step-2-ck-ccssa-form-6-2/ Page 4 of 70
Step 2 CK (CCSSA) Form 7 – Step Prep 30/10/21, 12:51 AM

12. A 37-year-old man comes to the physician because of progressive…

Demyelination of axons

Young woman with a recent URI who presents with ascending weakness, absent
deep tendon reflexes and mildly decreased touch/vibration, most consistent with
Guillain-Barre syndrome
Key idea: Although Guillain-Barre known for ascending weakness, it also commonly
leads to facial paralysis and/or respiratory failure
Key idea: Guillain-Barre is an autoimmune disease seen following a URI or GI
infection that leads to demyelination

13. Six hours after coronary artery bypass grafting, a 62-year-old…

Surgical exploration of the mediastinum

Patient <24 hours after CABG who develops hypotension, decreased urine output,
decreased cardiac output and a widened mediastinum, concerning for mediastinal
bleed
Key idea: Post-op patients with acute hemodynamic instability almost always will
require surgical exploration/correction (especially in NBME questions)

14. A 72-year-old man comes to the physician because of a…

Muscle

Older patient on a statin drug who presents with muscle tenderness and objective
muscle weakness, most consistent with a statin myopathy
Key idea: Important causes of a myopathy on NBME exam include
glucocorticoids/Cushing’s, inflammatory myopathies (polymyositis,
dermatomyositis), statin-induced myopathy and hypothyroid myopathy

15. A 42-year-old man comes to the emergency department at…

Discharge and encourage fluid intake

https://step-prep.org/step-2-ck-ccssa-form-6-2/ Page 5 of 70
Step 2 CK (CCSSA) Form 7 – Step Prep 30/10/21, 12:51 AM

Young, otherwise healthy man presents with severe flank pain, CVA tenderness and
hematuria with pain that is responsive to morphine, most consistent with kidney
stone
Key idea: Patients with a kidney stone that is <10 mm and/or not causing urosepsis
or renal failure can often be medically managed with hydration, pain control, alpha
blockers and straining urine (to look for passage of stone)

16. A 62-year-old woman is admitted to the hospital because…

Intravenous fluid therapy

Middle-aged woman with CKD (likely due to type 2 diabetes) who presents with
symptoms (fever, cough, abnormal CXR) concerning for pulmonary embolism vs
community-acquired pneumonia (fever, CXR finding) who also has poor oral intake
during her hospital stay; during the course of her hospital stay she receives a CT
angiogram to work-up potential pulmonary embolism and is later found to have an
acute kidney injury most likely due to contrast nephropathy (form of acute
tubular necrosis) in the setting of low fluid intake
Key idea: Contrast nephropathy is a feared complication of imaging studies with
contrast, particularly among patients with baseline renal insufficiency (such as this
patient), and many studies have found that IV fluid administration prior to contrast
administration can be useful for decreasing the risk of developing contrast
nephropathy
https://www.ncbi.nlm.nih.gov/pmc/articles/PMC5954945/

17. A previously healthy 42-year-old woman has had generalized…

Thymoma

Young woman + weakness affecting the eyes + anterior mediastinal mass =


myasthenia gravis secondary to thymoma
Key idea: Myasthenia gravis associated with thymoma, whereas Lambert-Eaton
associated with small cell lung cancer

18. An 18-year-old primigravid woman at 37 weeks’ gestation is…

https://step-prep.org/step-2-ck-ccssa-form-6-2/ Page 6 of 70
Step 2 CK (CCSSA) Form 7 – Step Prep 30/10/21, 12:51 AM

Amniotomy and vaginal delivery

Woman at 37 weeks’ gestation presenting in labor who has history of genital herpes
but currently does not have lesions or prodromal symptoms and therefore can
undergo vaginal delivery
Key idea: Women with history of HSV should begin acyclovir prophylaxis at 36
weeks gestation and women with active HSV lesions or prodromal symptoms
during labor should have a Cesarean delivery performed

19. A county health officer investigates an outbreak of illness…

Inadequate refrigeration of the implicated food

Causes of rapid-onset food poison (< 6 hours) are due to ingestion of pre-formed
toxin and are most commonly caused by Staph aureus (cream-based food such as
egg salad) or Bacillus cereus (reheated rice syndrome)
Key idea: Staph aureus food poisoning (most likely in this case given egg salad and
rapid-onset symptoms) can either be due to improper cooking or inadequate
refrigeration, but in this case given the history of a family picnic and the food likely
being left out all day, inadequate refrigeration is more likely

20. A 52-year-old man comes to the physician because of excruciating…

Indomethacin

Middle-aged man presents with acute onset, atraumatic pain of the great toe with
swelling, erythema and tenderness of the metatarsophalangeal joint on exam,
most consistent with acute gout
Key idea: Acute gout should be treated with NSAIDs (often indomethacin) and
patients should NOT be started on chronic gout drugs (allopurinol, probenecid, etc.)
until the acute flare has resolved because these drugs can lead to rapid shifts in uric
acid levels that can exacerbate/cause a new flare
Note: Patients can sometimes be given oral or intra-articular steroids, but NSAIDs
are tried first due to lower risk and good effectiveness in the majority of patients

https://step-prep.org/step-2-ck-ccssa-form-6-2/ Page 7 of 70
Step 2 CK (CCSSA) Form 7 – Step Prep 30/10/21, 12:51 AM

21. A 22-year-old primigravid woman at term is admitted to the…

Urinary stasis

As the uterus enlarges, it can begin to press upon nearby structures, including the
urethra –> Impaired bladder emptying –> urinary stasis –> nidus for infection

22. A 6-month-old boy is brought to the physician because of a…

Mupirocin ointment

Key idea: Honey-colored crust = Impetigo = Staph or Strep infection = Topical


antibiotic treatment (mupirocin)
Irritant dermatitis (erythema in the groin sparing flexural creases) –> Petroleum jelly
or zinc oxide
Candida dermatitis (beefy-red rash involving skinfolds and with satellite lesions) –>
Topical antifungal therapy (like nystatin)

23. A 4-month-old infant is brought to the emergency department…

Child abuse

Common injuries highly associated with child abuse include retinal hemorrhages
and intracranial injury (particularly subdural hematoma due to shearing of
bridging veins with violent shaking)
Can also commonly see long-bone spiral fractures or rib fractures, but importantly
the NBME will at times try to trick you by having a parent bring in an infant with a
history incompatible with bone fractures who has osteogenesis imperfecta
Key idea: One of the other presentations is if the parent’s story does not fit known
developmental milestones (saying that a 2 month old rolled off a table when children
typically do not roll until ~4 months)

24. A homeless 66-year-old man is admitted to the hospital because…

Decreased calcium

https://step-prep.org/step-2-ck-ccssa-form-6-2/ Page 8 of 70
Step 2 CK (CCSSA) Form 7 – Step Prep 30/10/21, 12:51 AM

Key idea: ON THE BOARDS, ALCOHOLISM = MALNUTRITION!


Hypomagnesemia –> Refractory hypokalemia and hypocalcemia
Key idea: In the setting of hypocalcemia, important to always check a magnesium
level and a serum albumin level (low albumin –> low total calcium)

25. A 16-year-old girl is brought to the physician by her mother…

Family therapy

Family therapy is appropriate in this case because the father, mother and daughter
could benefit from improved communication and coping strategies
Note: Family therapy is also effective in the management of a patient with
schizophrenia (helps to educate family about symptoms, course, etc.)

26. A 47-year-old woman is brought to the emergency department…

Thoracic aorta

Patient involved in a significant motor vehicle accident with acceleration-


deceleration forces who develops neck pain and chest pain with bruising over the
sternum and a widened mediastinum (almost always associated with aortic
dissection or aortic injury), most consistent with blunt thoracic aortic injury leading
to incomplete aortic rupture
Key idea: Involves the thoracic aorta because the aortic isthmus is tethered by the
ligamentum arteriosum and is therefore not freely mobile and is prone to
developing severe stretching/shearing forces in the setting of an acceleration-
deceleration injury
https://www.nejm.org/doi/full/10.1056/NEJMra0706159

27. A 52-year-old woman is brought to the physician by her…

Bipolar disorder

Patient with a history of episodes consistent with depression (sleep changes,


decreased energy/interest) who now presents with irritability, decreased need for

https://step-prep.org/step-2-ck-ccssa-form-6-2/ Page 9 of 70
Step 2 CK (CCSSA) Form 7 – Step Prep 30/10/21, 12:51 AM

sleep, pressured speech and distractibility, all consistent with bipolar disorder
Bipolar disorder associated with DIG FAST symptoms (Distractibility, Impulsivity,
Grandiosity, Flight of ideas, increased Activity, decreased Sleep and Talking
loudly/rapidly)
Key idea: Aspects of psychosis (such as this patient’s auditory hallucinations) can be
seen in multiple psychiatric conditions including bipolar disorder and are NOT
specific to schizophrenia, schizophreniform, etc.

28. One day after removal of a large meningioma, a 42-year-old…

Diabetes insipidus

Patient with recent brain surgery who has developed hypernatremia with dilute
urine and polyuria, consistent with diabetes insipidus (most likely central due to
brain surgery)
Causes of central diabetes insipidus (decreased ADH release): Pituitary tumor,
trauma, surgery, autoimmune disease, ischemic event
Causes of Nephrogenic diabetes insipidus (decreased responsiveness of kidney to
ADH): Inherited, hypocalcemia, hypokalemia, lithium, demeclocycline
Note: Excessive ADH production and renal salt wasting would lead to hyponatremia,
0.9% is isotonic (so wouldn’t lead to significant electrolyte changes), and
hyperaldosteronism leads to normal Na+ levels due to aldosterone escape

29. An asymptomatic 23-year-old man comes to the physician for…

Treatment of latent tuberculosis now

Patient with a positive PPD (>15 mm positive in all patients) who is asymptomatic
and without CXR findings = Latent tuberculosis –> Treatment (especially on NBME
exam)
Key idea: PPD induration (NOT erythema) required for positivity depends upon
patient risk factors with >5 mm being positive in patients with significant
immunosuppression (HIV, organ transplant, immunosuppressant meds), recent
contact with patient with active TB or patients with CXR findings consistent with
TB // >10 mm being positive for patients from Tb endemic countries, IVDU,

https://step-prep.org/step-2-ck-ccssa-form-6-2/ Page 10 of 70
Step 2 CK (CCSSA) Form 7 – Step Prep 30/10/21, 12:51 AM

residents of high-risk settings (prisons, nursing homes, homeless shelters, etc.),


children < 4 years old and patients working in mycobacterial labs // >15 mm in all
patients
Note: 3 treatment options for latent tuberculosis include (1) Isoniazid and rifapentine
+/- pyridoxine (2) Isoniazid +/- pyridoxine (3) Rifampin

30. A 24-year-old man has shortness of breath 2 days after open…

Fat embolism syndrome

Key idea: Fat embolism often develops 24 to 72 hours after inciting event (long-
bone fracture, orthopedic surgery, etc.)
Key idea: Fat embolism is a clinical diagnosis that leads to triad of (1) Respiratory
distress (2) Neurologic dysfunction (3) Petechial rash (or thrombocytopenia)

31. A previously healthy 6-year-old boy is brought to the…

Operative procedure

Young child presenting with acute unilateral scrotal pain, nausea/vomiting,


abdominal pain with involuntary guarding and a discolored, swollen hemiscrotum
prompting concern for testicular torsion which should be taken directly to OR!
Key idea: Surgical correction should be performed within 6 hours in order to salvage
the affected testicle and importantly torsion often occurs in the setting of an
inherited defect in the attachment of the testicle within the scrotum that will
affect both testicles, so an orchiopexy should be performed both on the affected
and unaffected side

32. A 47-year-old woman comes to the physician because of…

Intravascular volume depletion

Middle-aged woman with potential infectious disease (fever) leading to one week of
uncontrolled hyperglycemia who has signs of orthostatic hypotension (drop in
systolic BP >20 or diastolic BP >10 when moving from sitting to standing) and is

https://step-prep.org/step-2-ck-ccssa-form-6-2/ Page 11 of 70
Step 2 CK (CCSSA) Form 7 – Step Prep 30/10/21, 12:51 AM

found to have glucosuria, most consistent with intravascular volume depletion due
to glucose acting as an osmotic agent and leading to polyuria
Key idea: Infections often lead to increased insulin requirement by causing a
reactionary hyperglycemia

33. A 50-year-old woman with a 5-year history of metastatic breast…

Pericardial window

Middle-aged patient with a history of metastatic breast cancer who presents with
triad of hypotension, jugular venous distention and distant heart sounds who also
has electrical alternans and pericardial effusion, all consistent with a diagnosis of
pericardial tamponade often secondary to breast cancer metastasis to the
pericardium
Key idea: Although cardiac tamponade is often confirmed with an echocardiogram, if
your index of suspicion is high you would proceed directly to therapeutic pericardial
window due to significant morbidity/mortality associated with condition

34. A 4-year-old boy has had increasing fatigue since a viral illness…

Bone marrow aspiration

Young child with a recent URI who is presenting with pancytopenia and
lymphadenopathy and hepatomegaly, concerning for acute lymphoblastic
leukemia
Key idea: Viral infections are known for causing pancytopenia, but in this patient who
fits the correct demographic and has lymphadenopathy and hepatomegaly (signs of
lymphoid hyperplasia) fit well with a diagnosis of acute lymphoblastic leukemia,
which would have at least 20% blast cells on bone marrow aspiration

35. A 27-year-old woman comes to the physician because of a…

Small bowel obstrution

Young woman with Crohn’s disease who has severe abdominal pain,

https://step-prep.org/step-2-ck-ccssa-form-6-2/ Page 12 of 70
Step 2 CK (CCSSA) Form 7 – Step Prep 30/10/21, 12:51 AM

nausea/vomiting, and a distended tender abdomen, most consistent with small


bowel obstruction likely due to a stricture in setting of Crohn’s disease
Key idea: Elevated amylase is NOT specific for pancreatitis and can also be seen in
patient with vomiting
Key idea: Small bowel obstruction classically leads to 5 symptoms: (1) Obstipation
(constipation, no flatulence) (2) High-pitched bowel sounds (bowel working hard to
try to get past obstuction) (3) Abdominal distention (4) Abdominal pain (5)
Nausea/vomiting
Crohn’s disease leads to transmural fibrosis and can therefore damage underlying
stem cells and lead to fibrotic strictures through healing process

36. An 82-year-old woman with metastatic breast cancer who is…

Increase the frequency of dose to every 3 hours

Key idea: While physicians should be on the look-out for signs of opioid addiction in
patients receiving pain medications, in patients with metastatic cancer to bone it is
common for them to develop tolerance and for the pain to progress, so their
requests for increased pain management are valid and should be managed through
a palliative care lens
Mild cancer pain: Acetaminophen, NSAIDs
Moderate cancer pain: Weak opioids (codeine, hydrocodone, tramadol)
Severe cancer pain: Strong short-acting opioids (morphine, hydromorphone) –>
Long-acting opioids (fentanyl, oxycodone) and short-acting opioids for breakthrough
pain

37. Two days after beginning ACTH therapy for multiple sclerosis…

Haloperidol

Key idea: Systemic steroids are associated with psychosis and this patient is
receiving ACTH, which will stimulate the adrenal gland to release increased
amounts of glucocorticoids/steroids, likely precipitating her psychosis (delusions,
bizarre behaviors, etc.)
Alprazolam = Benzo (avoided in older patients due to risk of delirium)

https://step-prep.org/step-2-ck-ccssa-form-6-2/ Page 13 of 70
Step 2 CK (CCSSA) Form 7 – Step Prep 30/10/21, 12:51 AM

Amitriptyline = TCA antidepressant (refractory depression, neuropathic pain,


migraines)
Lithium = Bipolar disorder
Sertraline = SSRI (depression, PTSD, panic disorder, etc.)

38. A 27-year-old man comes to the physician with his 27…

Bilateral varicoceles

Key idea: Varicoceles are a major cause of male-dependent infertility because the
increased pooling of blood in the pampiniform plexus near the testes leads to
increased temperature, which leads to impaired Sertoli cell function (Note: Leydig
cells will still be functioning well so testosterone levels will be normal)
Key idea: In NBME-style infertility questions, if one of the partners has had a
previous child then the problem likely involves the other individual

39. A previously healthy 4-year-old boy is brought to the physician…

Upper respiratory tract infection

Did not have access to media player, but the patient’s presentation is consistent with
a URI
CHF: Cyanosis, delayed capillary refill, etc.
Idiopathic pulmonary hypertension: Lower extremity edema, congestive
hepatopathy, loud P2, RV heave
Pericarditis: Febrile, friction rub, etc.
VSD: Often asymptomatic until later age, will lead to a systolic ejection murmur in the
lower left sternal border

40. A previously healthy 26-year-old man comes to the…

Increased calcium

Young man presents with fever, erythema nodosum (painful, red, tender nodules on
anterior shins) and bilateral hilar fullness, most consistent with sarcoidosis

https://step-prep.org/step-2-ck-ccssa-form-6-2/ Page 14 of 70
Step 2 CK (CCSSA) Form 7 – Step Prep 30/10/21, 12:51 AM

Key idea: Sarcoidosis (like other forms of granulomatous disease) is associated with
hypercalcemia because of increased activity of 1-alpha hydroxylase leading to
increased production of activated vitamin D –> hypercalcemia,
hyperphosphatemia
Ddx for erythema nodosum: Sarcoidosis, Strep infections, tuberculosis,
inflammatory bowel disease, Behcet disease, endemic fungal disease (cocci, histo)
Key idea: If patient has erythema affecting both legs, then cellulitis is virtually ruled
out! (bilateral cellulitis very rare)

41. A 4-year-old girl is brought to the physician because of cough…

T-lymphocyte dysfunction

Young child with non-reactive skin testing for multiple antigens (which is a type 4
hypersensitivity process dependent upon T cells), raising concern for T
lymphocyte dysfunction
Key idea: Patients with HIV can have a falsely-negative PPD in the setting of M.
tuberculosis because they have low T cell function and impaired Type 4 HS
reaction
Antibody deficiency = Bruton’s agammaglobulinemia = Recurrent infections with
encapsulated infections
Complement deficiency = Recurrent Neisseria infections
Impaired chemotaxis = Leukocyte adhesion deficiency = Recurrent abscesses
without pus, delayed wound healing, delayed umbilical cord separation
Impaired respiratory burst = chronic granulomatous disease = Recurrent skin
abscesses and infections with catalase positive organisms (Staph, Pseudomonas,
Aspergillus, Nocardia, Serratia, etc.)
Splenic dysfunction (sickle cell disease, trauma patient) = Encapsulated infections
(Strep pneumo, H. influenzae, Neisseria)

42. A 47-year-old woman comes to the physician because of fever…

CT scan of the head

Patient presenting with signs/symptoms concerning for meningitis vs.

https://step-prep.org/step-2-ck-ccssa-form-6-2/ Page 15 of 70
Step 2 CK (CCSSA) Form 7 – Step Prep 30/10/21, 12:51 AM

subarachnoid hemorrhage who also has signs of increased intracranial pressure


and should therefore have a CT scan performed before a lumbar puncture
Key idea: Optic fundi cannot be visualized = Papilledema = Increased intracranial
pressure = Contraindication to lumbar puncture due to risk of cerebral herniation
Key idea: Both meningitis and subarachnoid hemorrhage can lead to fever and signs
of meningismus (pain with neck flexion, Kernig sign, etc.)

43. A 24-year-old woman comes to the physician because she has…

Osteoporosis

Anorexia nervosa –> Functional hypothalamic amenorrhea (also seen in extreme


exercisers, professional athletes, etc.) –> Decreased GnRH release from
hypothalamus –> Decreased LH and FSH release from anterior pituitary –>
Decreased estrogen production by ovaries –> Increased osteoclast activity and
decreased osteoblast activity –> Osteoporosis

44. A 19-year-old man is brought to the emergency department by…

PCP

Patients with PCP intoxication often have nystagmus, violent behavior and
fluctuations between irritability and severely decreased activity (mute,
motionless, etc.)
Key idea: On the NBME, drug intoxication + nystagmus = PCP use

45. An 82-year-old woman is brought to the emergency department…

Central retinal vein occlusion

Unilateral vision loss + Dilated retinal veins and widespread retinal hemorrhages on
fundoscopy (“blood and thunder” appearance) = Central retinal vein occlusion
Central retinal artery occlusion = Cherry-red spot on fundoscopy
Corneal abrasion = Positive Fluorescein staining, sensitivity to light, pain
Glaucoma = Bilateral lens opacity and glare from lights

https://step-prep.org/step-2-ck-ccssa-form-6-2/ Page 16 of 70
Step 2 CK (CCSSA) Form 7 – Step Prep 30/10/21, 12:51 AM

HSV conjunctivitis = Dendritic ulcers


Optic neuritis = Optic disk pallor

46. A 37-year-old woman has had malaise, muscle aches…

Culture for herpes simplex

Painful genital ulcers: (1) Herpes simplex virus: Small vesicles/ulcers on


erythematous base with mild lymphadenopathy (2) Chancroid: Large, deep ulcers
with a soft, friable base and severe suppurative lymphadenopathy
Painless genital ulcers: (1) Syphilis: Single ulcer with regular borders and hard base
(chancre) (2) Lymphogranuloma venereum: Painless, shallow ulcers with painful
fluctuant lymphadenopathy (buboes)

Exam section 2:

1: Over the past 2 years, a 67-year-old man has had gradually…

Abstinence from alcohol

Older man with significant smoking and drinking history who presents with signs of
cerebellar dysfunction (ataxia, wide-based gait), most consistent with alcoholic
cerebellar degeneration (degeneration of Purkinje cells in cerebellar vermis –>
Truncal dyscoordination with impaired gait and postural incoordination with
preserved limb coordination)

2. A 27-year-old male physician is inadvertently struck with a…

No treatment necessary

Patient is adequately vaccinated against hepatitis B (“adequate concentrations of


antihepatitis B antibodies) and therefore does not need post-exposure prophylaxis
Key idea: If patient was unvaccinated or had unknown vaccination history, they would
receive both hep B immunoglobulin and hepatitis B vaccine

3. A previously healthy 32-year-old woman at 16 weeks’ gestation…

https://step-prep.org/step-2-ck-ccssa-form-6-2/ Page 17 of 70
Step 2 CK (CCSSA) Form 7 – Step Prep 30/10/21, 12:51 AM

Thrombotic thrombocytopenic purpura

Young pregnant woman presenting with fever, neurologic dysfunction, anemia with
schistocytes (microangiopathic hemolytic anemia), and thrombocytopenia,
associated with thrombotic thrombocytopenia purpura
Mnemonic for components of thrombotic thrombocytopenic purpura or hemolytic-
uremic syndrome: Brain FART –> Brain (neuro symptoms, more associated with
TTP), Fever, Anemia (hemolytic), Renal problems (more associated with HUS),
Thrombocytopenia
Causes of micro/macroangiopathic hemolytic anemia (Anemia with schistocytes,
unconjugated hyperbilirubinemia, etc.): DIC, TTP, HUS, SLE, HELLP syndrome,
hypertensive emergency, aortic stenosis, prosthetic heart valves
Henoch-Schonlein purpura: SAG –> Skin (palpable purpura), Arthralgias and GI
symptoms (with association with intussusception)

4. Six hours after undergoing left thoracostomy for a coarctation…

Analgesic therapy

Newborn who underwent major cardiac surgery who is fussy/irritable with mildly
elevated pulse and mildly increased blood pressure, which is most likely due to
inadequate pain management

5. A 43-year-old man comes to the physician because of a 3-day…

Haemophilus influenzae

Middle-aged man with significant smoking history and signs of pneumonia (fever,
pleuritic chest pain, productive cough, shortness of breath, increased tactile fremitus
and dullness to percussion) who has sputum gram stain showing gram-negative
bacilli, most consistent with H. influenzae
Pseudomonas aeruginosa would also lead to gram-negative bacilli, but would be
more common in a cystic fibrosis patient
N. meningitidus –> Gram-negative cocci
Strep pneumo –> Gram positive diplococci

https://step-prep.org/step-2-ck-ccssa-form-6-2/ Page 18 of 70
Step 2 CK (CCSSA) Form 7 – Step Prep 30/10/21, 12:51 AM

6. A previously healthy 57-year-old man comes to the emergency…

Increased urinary excretion of calcium

Middle-aged man with signs/symptoms and CT scan consistent with renal calculi
who has lab findings consistent with primary hyperparathyroidism (increased
calcium, decreased phosphorous, increased PTH)
Key idea: Although elevated PTH levels will lead to increased reabsorption of calcium
from the renal tubules, the amount of calcium being filtered by the kidney will still be
relatively higher than normal due to increased calcium in the blood

7. A 67-year-old man comes to the emergency department because…

No cleaving of C-terminal peptides on angiotensin 1

Angiotensinogen –(renin)–> Angiotensin 1 –(angiotensin-converting enzyme)–>


Angiotensin 2 –> Vasoconstriction, increased aldosterone, increased ADH, increased
sympathetic activity
Key idea: Although patient on ACE inhibitor will have decreased mineralocorticoid
production in the adrenal cortex, the decreased angiotensin 2 will have a larger
overall effect

8. A 32-year-old man is brought to the emergency department…

Hypertrophic obstructive cardiomyopathy

Young man with exertional syncope who has a systolic ejection murmur at the left
sternal border that increases with decreased preload –> HOCM
Key idea: Only 2 heart murmurs that increase in intensity with decreased preload
(sitting to standing, diuretics, etc.) are HOCM (decreased preload –> increased
obstruction) and Mitral valve prolapse (decreased preload –> less taut chordae
tendinae –> earlier prolapse)

9. A 22-year-old man is brought to the emergency department…

Atropine

https://step-prep.org/step-2-ck-ccssa-form-6-2/ Page 19 of 70
Step 2 CK (CCSSA) Form 7 – Step Prep 30/10/21, 12:51 AM

Patient exposed to gas who presents with signs of organophosphate poisoning


(increased Ach leads to muscle fasciculations, increased sweating and increased
parasympathetic effects (miosis, drooling, rhinorrhea, bronchoconstriction)
Organophosphates are acetylcholinesterase inhibitors –> Increased acetylcholine
and therefore can be treated by using muscarinic antagonists (such as atropine)
Key idea: Atropine will block the muscarinic effects of increased acetylcholine, but
will not block the nicotinic effects (muscle fasciculation –> muscle paralysis)
Atropine = Muscarinic antagonist –> Increased body temperature (decreased
sweating), decreased parasympathetic functions (tachycardia, dry mouth, mydriasis,
constipation, disorientation, urinary retention)

10. A 32-year-old woman comes to the physician because of…

Amoxicillin

Young patient with headache, arthritis, fever several weeks after a red rash who has
IgM antibodies (indicator of recent infection) positive for Borrelia burgdorferi, most
consistent with Lyme disease
Lyme disease often treated with doxycycline, but in pregnant women and children it
is often treated with amoxicillin or cefuroxime (due to teratogenic effects of
doxycycline)

11. A 57-year-old woman comes to the physician because of…

Osteoarthritis

Older woman without signs of inflammation (no fever, normal ESR) who has
asymmetric groin pain (indicative of hip pathology) with an x-ray showing joint-
space narrowing particularly between the left femoral head and acetabulum,
consistent with osteoarthritis
Pathology in the hip –> Groin pain (HIGH YIELD!!!!)
Features of osteoarthritis –> Joint cysts/osteophyte, subchondral sclerosis, joint-
space narrowing

12. A 54-year-old man comes to the physician because of weakness…

https://step-prep.org/step-2-ck-ccssa-form-6-2/ Page 20 of 70
Step 2 CK (CCSSA) Form 7 – Step Prep 30/10/21, 12:51 AM

Carotid endarterectomy

Carotid endarterectomy should be considered in patients who are symptomatic (TIA


or ischemic stroke in distribution of affected vessel) with high-grade carotid
stenosis (70-99%) without relative contraindications (persistently disabling
neurologic deficits, life expectancy < 5 years, etc.)
Key idea: Carotid endarterectomy does NOT reverse the damage from the stroke
that has already occurred, but would decrease risk of further strokes/TIA
Key idea: All patients with carotid artery stenosis should be initially managed with
intensive medical management (aspirin, statin, BP control) and lifestyle interventions
(quitting smoking, weight loss, etc.)

13. An asymptomatic 22-year-old nulligravid woman comes to the…

Oral contraceptive therapy

Benefits of OCPs: Contraceptive, decreased endometrial and ovarian cancer,


treatment of hyperandrogenism (PCOS, acne)
Risks of OCPs: DVTs, hypertension, hepatic adenoma, stroke, increased risk of
cervical cancer
Key idea: No effective screening (imaging, lab test) is routinely available for ovarian
cancer

14. A sexually active 24-year-old man has had increasingly…

Genital herpes

“Dew drops on rose petals” appearance = Herpes infection


Painful genital ulcers: (1) Herpes simplex virus: Small vesicles/ulcers on
erythematous base with mild lymphadenopathy (2) Chancroid: Large, deep ulcers
with a soft, friable base and severe suppurative lymphadenopathy

15. A 12-month-old boy is brought to the emergency department…

B lymphocyte

https://step-prep.org/step-2-ck-ccssa-form-6-2/ Page 21 of 70
Step 2 CK (CCSSA) Form 7 – Step Prep 30/10/21, 12:51 AM

Young boy who has been infected with Strep pneumo, H. influenzae, and now is
infected N. meningitidis, all of which are the classic encapsulated bacteria, most
consistent with X-linked Bruton’s agammaglobulinemia
Key idea: Encapsulated bacteria (SHiN: Strep pneumo, H. Influenzae and Neisseria)
are harder for our bodies to destroy and are therefore more dependent upon
opsonization, which requires IgG (which will be absent in patients with defects in B
lymphocytes because they can’t produce plasma cells and therefore can’t
produce immunoglobulins)

16. A 12-month-old boy is brought to the physician because of a…

Neutrophil

Young boy with persistent Staph aureus skin abscesses and lung empyema (which is
commonly caused by Staph aureus), most consistent with chronic granulomatous
disease (X-linked disease most commonly)
Key idea: Patients with chronic granulomatous disease are preferentially infected by
catalase positive organisms, which can be remembered with the mnemonic BELCH
SPANS (Burkholderia cepacia, E. coli, Listeria, Candida, H. pylori, Staph aureus,
Pseudomonas aeruginosa, Aspergillus, Nocardia, Serratia)

17. A previously healthy 57-year-old woman with hypertension is…

Botulism

Previously healthy middle-aged woman who had home-canned foods who presents
with GI symptoms, ocular symptoms, and autonomic symptoms with normal deep
tendon reflexes, most consistent with Botulism
Key idea: Adult-onset botulism associated with consumption of canned foods,
whereas infant-onset botulism (floppy baby syndrome) associated with consumption
of honey and exposure to dust (construction site, etc.)
Guillain-Barre syndrome –> Ascending weakness over course of weeks associated
with sensory symptoms and loss of deep tendon reflexes
Myasthenia gravis –> Often does not present acutely, would not present with
autonomic symptoms and history often includes weakness that worsens with

https://step-prep.org/step-2-ck-ccssa-form-6-2/ Page 22 of 70
Step 2 CK (CCSSA) Form 7 – Step Prep 30/10/21, 12:51 AM

continued use, associated with thymoma


Lambert-Eaton syndrome –> Often does not present acutely, presents with
autonomic dysfunction and diminished/absent deep tendon reflexes, and improves
with continued use, associated with small cell lung cancer

18. A 36-year-old woman, gravida 1, para 1, has had heavy vaginal…

Uterine atony

Woman with complicated/prolonged delivery with excessive postpartum bleeding


who does not have retained placenta, genital tract trauma or history of
coagulopathy, and therefore most likely has uterine atony (most common cause of
postpartum bleeding and commonly seen after prolonged/difficult delivery)
Causes of post-delivery bleeding is 4 T’s: (1) aTony (most common) (2) Trauma (3)
Thrombin (coagulopathy) (4) Tissue (retained placenta, etc.)

19. A 25-year-old primigravid woman at 27 weeks’ gestation is…

Intramuscular administration of betamethasone

Patient presenting with preterm labor < 32 weeks: (1) Betamethasone (for fetal
lung development) (2) IV penicillin (if GBS positive or unknown) (3) Magnesium
(neuroprotective for baby) (4) Tocolytic (to stop contractions)
Active labor: Contractions AND cervical changes
C-section would only be indicated if there were signs of intramniotic infection, fetal
instability or maternal instability
Note: Patients should receive intravenous antibiotics rather than oral antibiotics in a
patient who is GBS positive or GBS unknown

20. A previously healthy 27-year-old man is brought to the…

Vasovagal syncope

Young otherwise healthy man who experienced loss of conciousness preceded by


prodromal symptoms (nausea, sweating, lightheadedness) in a stressful situation

https://step-prep.org/step-2-ck-ccssa-form-6-2/ Page 23 of 70
Step 2 CK (CCSSA) Form 7 – Step Prep 30/10/21, 12:51 AM

and was found to be bradycardic at the time of the loss of consciousness, most
consistent with vasovagal syncope
Key idea: Vasovagal syncope is the most common cause of loss of consciousness,
especially among otherwise healthy patients
Hypoglycemia can also lead to loss of consciousness among otherwise healthy
patients, but would not likely be associated with bradycardia or prodrome

21. A 32-year-old nulligravid woman with polycystic ovarian syndrome…

Clomiphene

PCOS patient who does not wish to become pregnant: Dual estrogen-progestin
OCPs
PCOS patient who wants to become pregnant: Clomiphene (antagonists at
estrogen receptors in hypothalamus –> prevents normal feedback inhibition –>
increased release of LH and FSH from pituitary –> stimulates ovulation)
Note: PCOS patients with all types of fertility goals would benefit from weight loss

22. An afebrile 32-year-old woman comes to the physician because…

Enterotoxic Escheria coli

Pathogens leading to non-inflammatory diarrhea with no fecal leukocytes: V.


cholerae, ETEC, B. cereus, Staph aureus, Giardia, etc.
Staph aureus = Acute onset food poisoning that resolves within 24 hours
Shigella and Yersinia = Bloody diarrhea with fecal PMNs

23. A 37-year-old man with end-stage Duchenne muscular dystrophy…

Determine if the patient wants to discuss his decision with anyone else

Key idea: Normal for patients with terminal illness to feel sad about their situation,
but also important to look out for full-blown major depressive disorder
Should not force patient to discuss his plans with family, but important to determine
whether patient would like other important stakeholders to be involved in the

https://step-prep.org/step-2-ck-ccssa-form-6-2/ Page 24 of 70
Step 2 CK (CCSSA) Form 7 – Step Prep 30/10/21, 12:51 AM

discussion

24. A 57-year-old woman with factor XI and IgA deficiencies is…

Anaphylactic transfusion reaction

Key idea: IgA deficiency classically leads to Anaphylaxis to IgA-containing blood


products + Atopy + Autoimmune disease + GI/sinopulmonary infections (IgA
involved in mucosal immunity)
Anaphylactic transfusion reaction: Respiratory distress/wheezing, angioedema,
hypotension, hives/urticaria
Urticarial transfusion reaction: Angioedema, wheezing

25. A previously healthy 4-year-old boy is brought to the emergency…

Supination of the forearm with the elbow in slight flexion

Young patient with atraumatic elbow pain holding extremity with elbow flexion and
forearm in pronation with no obvious signs of injury, which is most consistent with a
Radial head subluxation (Nursemaid’s elbow) which commonly occurs when young
children have their arm pulled and the radial head gets trapped beneath the annular
ligament
Key idea: Can be treated by hyperpronation of forearm or by supination of forearm
with flexion of elbow

26. A 7-year-old boy is brought for an examination prior to a…

Viridans streptococci

Key idea: Patients with abnormal heart valves are prone to developing endocarditis
during periods of bacteremia, with dental work and toothbrushing leading to
transient bacteremia with Viridans streptococci
Key idea: Staph aureus (IVDU, bacteremia) leads to acute endocarditis (days),
whereas Viridans streptococci (dental work) leads to subacute endocarditis (weeks)

27. A 50-year-old woman with a history of hypothyroidism has…

https://step-prep.org/step-2-ck-ccssa-form-6-2/ Page 25 of 70
Step 2 CK (CCSSA) Form 7 – Step Prep 30/10/21, 12:51 AM

Atrophic gastritis

Middle-aged woman with history of autoimmune disease who presents with


megaloblastic anemia + neurologic symptoms with an abnormal Schilling test
corrected by administration of intrinsic factor, most consistent with Autoimmune
gastritis (antibodies against parietal cells in the stomach lead to (1) impaired intrinsic
factor production –> B12 deficiency (2) chronic inflammation –> chronic/atrophic
gastritis
Key idea: B12 deficiency leads to subacute combined degeneration, which often
leads to problems with strength, fine touch, vibration and balance/cerebellar
functions

https://www.memorangapp.com/flashcards/130443/USMLERx/

28. A previously healthy 47-year-old man comes to the physician…

Fibrillation potentials in multiple muscles of multiple extremities

Young man with chronic weakness who has UMN lesions (Babinski sign) and LMN

https://step-prep.org/step-2-ck-ccssa-form-6-2/ Page 26 of 70
Step 2 CK (CCSSA) Form 7 – Step Prep 30/10/21, 12:51 AM

lesions (atrophy, fibrillations), most consistent with amyotrophic lateral sclerosis


(ALS), which leads to fibrillation potential in multiple muscles on electromyography
Key idea: ALS often presents with weakness in hands, difficulty swallowing (like in
this patient), and changes to voice

29. An asymptomatic 27-year-old woman who is HIV positive…

Pneumococcal vaccine

Additional vaccination requirements for patients with HIV include (1) Vaccination for
hepatitis B unless they have documented immunity (2) Strep Pneumo PCV13
followed by the 23-valent PPSV23 8 weeks later and again in 5 years and at age
65 (3) Meningococcal vaccine with boosters every 5 years
Note: Patients should receive one Tdap vaccine as an adult followed by boosters
every 10 years

30. An 11-year-old girl is brought to the physician by her parents…

Education about puberty for the child and parents

Key idea: Patient is displaying normal teenager behavior and her symptoms are not
causing her distress or leading to societally-inappropriate behavior (causing
fires, stealing, etc.)

31. Two hours after emergency repair of a perforated ulcer, a…

Decreased potassium

Key idea: Digoxin toxicity can lead to premature PVCs (along with other
arrhythmias), with digoxin toxicity being more prevalent in setting of hypokalemia
(less potassium to compete with digoxin for binding to the Na/K ATPase)

32. A 55-year-old woman with known metastatic breast cancer…

Increased calcium

https://step-prep.org/step-2-ck-ccssa-form-6-2/ Page 27 of 70
Step 2 CK (CCSSA) Form 7 – Step Prep 30/10/21, 12:51 AM

Metastatic breast cancer –> Bone involvement –> Hypercalcemia –> Hypercalcemic
crisis –> Oliguria/anuria and mental status changes (somnolence or coma)
Key idea: Hypercalcemic crisis often only seen when calcium levels are > 14 mg/dL,
and calcium levels often do not reach levels > 14 mg/dL except for in setting of
malignancy

33. A 52-year-old woman comes to the emergency department…

Endoscopic retrograde cholangiopancreatography

Middle-aged woman with recent lap chole presenting with five days of fever,
jaundice, and RUQ tenderness with labs showing a leukocytosis with cholestatic
pattern (Alkaline phosphatase >> AST), most concerning for acute cholangitis that
needs to be rapidly evaluated with an ERCP (associated with significant
morbidity/mortality)
Key idea: Acute cholangitis –> Fever, jaundice, RUQ pain (Charcot’s triad) +
hypotension, altered mental status (Reynold’s pentad)
Key idea: Even though patient has had cholecystectomy, they can still develop acute
cholangitis due to a residual stone in the common bile duct or a retained gallstone
in the cystic duct stump

34. A 42-year-old woman comes to the physician because of…

Urethral diverticulum

Woman with urinary incontinence exclusively after voiding found to have a cystic,
tender mass in the vagina and a low post-void residual volume, suggestive of a
urethral diverticulum
Interstitial cystitis: Discomfort/pain in bladder + Urinary frequency/urgency
Vesicovaginal fistula: Continuous loss of urine with clear fluid found in the vagina

https://step-prep.org/step-2-ck-ccssa-form-6-2/ Page 28 of 70
Step 2 CK (CCSSA) Form 7 – Step Prep 30/10/21, 12:51 AM

https://www.youtube.com/watch?v=OsvxoMl62q8

35. An 8-hour-old newborn develops jaundice and respiratory…

Rh incompatibility

Newborn is found to have unconjugated hyperbilirubinemia with anemia (consistent


with hemolytic anemia) and signs of volume overload (hepatosplenomegaly, edema),
most consistent with Rh-hemolytic disease
Potential causes of anasarca or hydrops fetalis: Rh-hemolytic disease,
parvovirus/CMV infection of mother, hemoglobin Barts disease (severe form of alpha
thalassemia), etc.
Key idea: ABO incompatibility will not present with as severe of a presentation
because majority of maternal antibodies against other AABO blood groups are IgM
and do not readily cross the placenta (contrast with acquired Rh-targeting antibodies
in Rh-negative mother who has been exposed to Rh-positive blood which are IgG and
can easily cross the placenta)

36. A 32-year-old man comes to the physician because of a 3-month…

https://step-prep.org/step-2-ck-ccssa-form-6-2/ Page 29 of 70
Step 2 CK (CCSSA) Form 7 – Step Prep 30/10/21, 12:51 AM

Trial of omeprazole therapy

Key idea: Patients with symptoms of GERD who are under 50 years old with
symptoms for <5 years, no cancer risk factors and no alarm symptoms (melena,
hematemesis, weight loss, anemia, dysphagia/odynophagia, etc.) should receive a
trial of PPI therapy
Key idea: Patients who are 50+ years old with symptoms for >5 years or cancer risk
factors or alarm symptoms would first receive an endoscopy

37. Four days after undergoing a right hemicolectomy for cecal…

Wound dehiscence

Key idea: Wound dehiscence often occurs ~5 days post-op


Wound dehiscence: Surgical wound opens back up, leading to increased
serosanguinous discharge without signs of infection –> Manage conservatively with
regular dressing changes, limiting mobilization, coughing, etc.
Evisceration: Dehiscence with protrusion of abdominal organs (viscera) through
opening –> SURGICAL EMERGENCY!!!

38. A 37-year-old man comes to the physician because of a 6-month…

Alveolar hypoventilation

PAO2 = 150 – (PaCO2/0.8) = 150 – (56/0.8) = 150 – 70 = 80


A-a gradient = 80 – 70 = 10 < 12 –> Normal A-a gradient consistent with
hypoventilation or decreased FiO2 (high altitude
Key idea: Common causes of hypoventilation include neuromuscular weakness, drug
intoxication (particularly opioids) and obesity hypoventilation syndrome (which is
most likely the case here)

39. A 17-year-old boy is brought to the emergency department…

Naloxone

Young patient with known heroin use who presents with altered mental status and

https://step-prep.org/step-2-ck-ccssa-form-6-2/ Page 30 of 70
Step 2 CK (CCSSA) Form 7 – Step Prep 30/10/21, 12:51 AM

hypoventilation, most consistent with heroin overdose


Opioid overdose should be treated immediately with naloxone (short-acting opioid
receptor antagonist), and use medications such as naltrexone (long-acting opioid
antagonists), methadone (long-acting opiate) or buprenorphine (partial opioid
agonist) to prevent relapse
Flumazenil –> Benzodiazepine overdose

40. A 37-year-old woman with alcoholism is admitted to the…

Acute respiratory distress syndrome

Patient with severe pancreatitis who develops hypoxia, hypotension with elevated
right-sided pressures and normal left-sided pressures and a severely increased A-a
gradient, most consistent with acute respiratory distress syndrome
PAO2 = (713 x 0.6) – (PaCO2/0.8) = 428 – (38/0.8) = 428 – 48 = 380 >> 12 –>
Increased A-a gradient
Congestive heart failure –> Decreased cardiac index and increased PCWP
Causes of ARDS: Sepsis, aspiration, pneumonia, trauma, pancreatitis

41. A 15-year-old boy is brought to the physician by his mother…

Major depressive disorder

Patient with acute stressor who develops behavior changes with changes in sleep,
energy, appetite, interest, cognitive dysfunction with an abnormal affect, most
consistent with major depressive disorder
Depression –> Depressed mood and SIGECAPS (Sleep changes, decreased
Interest, Guilt, decreased Energy, Cognitive dysfunction, Appetite changes,
Psychomotor slowing, Suicidal

42. A 75-year-old man is brought to the emergency department…

Heat stroke

Heat stroke: Elevated temperature with CNS dysfunction, caused by impaired

https://step-prep.org/step-2-ck-ccssa-form-6-2/ Page 31 of 70
Step 2 CK (CCSSA) Form 7 – Step Prep 30/10/21, 12:51 AM

thermoregulation
Heat exhaustion: Elevated temperature with no CNS dysfunction, caused
by inadequate salt and water replacement
Malignant hyperthermia –> Often seen in patient receiving anesthetic who
develops hyperthermia and rigidity

43. A 32-year-old man with major depressive disorder has had…

Methamphetamine

Methamphetamine is associated with a sensation of insects crawling on the skin –>


skin picking
Technical term is delusional parasitosis

44. A 62-year-old woman is admitted to the hospital for…

Renal ultrasonography

Middle-aged woman with a recent GU surgery who develops a significant AKI with a
BUN:creatinine ratio < 15 with a bland urinalysis, most concerning for a post-renal
AKI secondary to ureteral damage, which would be worked-up with a renal
ultrasound looking for hydronephrosis
Key idea: GU surgeries in women are highly associated with damage to the ureters,
which can lead to a post-renal AKI (VERY HIGH YIELD FOR NBME EXAMS!)

45. A 72-year-old man is admitted to the hospital because of shortness…

Diuretic therapy

Decompensated heart failure most likely diagnosis given demographics (older


patient with history of cardiac ischemia) + bilateral crackles in lung fields + low
cardiac index with an increased pulmonary artery occlusion pressure (which is
synonymous with pulmonary capillary wedge pressure and is a marker of left
atrial pressure
Decompensated heart failure –> Diuretic therapy to help pull off fluids (while being

https://step-prep.org/step-2-ck-ccssa-form-6-2/ Page 32 of 70
Step 2 CK (CCSSA) Form 7 – Step Prep 30/10/21, 12:51 AM

careful to not cause AKI)

46. A 17-year-old girl is brought to the emergency department…

Cricothyrotomy

Patient with massive facial trauma precluding standard tracheal intubation and
gurgling noises on exam, who should have an airway established via a
cricothyrotomy
Indications for cricothyrotomy: (1) Inability to intubate (2) Inability to ventilate (3)
Inability to maintain O2 sat > 90% (4) Severe traumatic injury that prevents
oral/nasal tracheal intubation
Key idea: Cricothyrotomy is most often used in patients who have experienced a
traumatic injury

Exam section 3:

1: A 34-year-old woman has had pain in the left hemithorax…

Costochondritis

Patient with history of increased activity + localized pain to the anterior chest
that worsens with movement, deep inspiration and cough + Tenderness to
palpation of affected area = Costochondritis
Key idea: Self-limiting over a matter of weeks, but may persist for up to a year

2. A 72-year-old man is hospitalized because of dyspnea for 6…

Cor pulmonale

Patient with progressive dyspnea with signs of right-sided heart failure (JVD,
ascites, pitting edema but no pulmonary edema) and a history of DVT/PE, most
consistent with cor pulmonale secondary to Group 4 pulmonary hypertension
Group 1 pulmonary hypertension = Pulmonary arterial hypertension (pre-capillary)
[familial, toxins, connective tissue disease, etc.]
Group 2 PH: Left-sided heart failure

https://step-prep.org/step-2-ck-ccssa-form-6-2/ Page 33 of 70
Step 2 CK (CCSSA) Form 7 – Step Prep 30/10/21, 12:51 AM

Group 3 PH: Chronic lung disease (COPD, OSA, asthma, etc.)


Group 4 PH: Chronic thromboembolic disease
Group 5 PH: Other causes [anemia, sarcoidosis, etc.]
Aortic stenosis or Mitral regurgitation = Systolic murmur on exam
Ischemic heart disease = Chest pain vs heart failure (pulmonary edema on chest x-
ray)
Viral cardiomyopathy = Acute dyspnea, chest pain, fever

3. For 12 hours, a 25-year-old woman, gravida 2, para 1, at 20 weeks’…

Measurement of serum amylase activity

Young woman with symptoms concerning for acute pancreatitis (severe epigastric
pain radiating to the back with nausea/vomiting)
Key idea: Measurement of serum amylase/lipase preferable to CT scan in setting of
clinical pancreatitis (severe epigastric pain radiating to the back) because there can
be a delay in the appearance of imaging findings
Diagnosis of acute pancreatitis requires 2/3: (1) Acute epigastric pain radiating to
the back (2) Increased serum amylase or lipase to at least 3X upper limit of normal
(3) Characteristic imaging findings

4. A 21-year-old primigravid woman at 37 weeks’ gestation has had…

Measurement of platelet count

Patient with signs of pre-eclampsia (hypertension and proteinuria) who should have
delivery of the baby (because patient at 37 weeks’ gestation) and work-up for
other end-organ complications associated with severe pre-eclampsia (platelet
count, BUN/creatinine, etc.)
Pre-eclampsia (uncomplicated –> delivery at 37 weeks) = New-onset elevated
blood pressure at >20 weeks gestation AND Proteinuria (Urine protein:creatinine
ratio > 0.3 or 24-hour urine collection > 300 mg) or signs of end-organ damage
Features that point to severe pre-eclampsia requiring delivery at 34 weeks: BP >
160/110, thrombocytopenia, pulmonary edema, visual/cerebral symptoms,
increased creatinine, increased LFTs

https://step-prep.org/step-2-ck-ccssa-form-6-2/ Page 34 of 70
Step 2 CK (CCSSA) Form 7 – Step Prep 30/10/21, 12:51 AM

5. A 32-year-old man comes to the physician with his wife because…

Acute stress disorder

Symptoms of Acute stress disorder or PTSD: Patient with exposure to life-


threatening trauma who experiences HARD symptoms (Hyperarousal, Avoidance
behaviors, Re-living experience, Disturbed mood)
Acute stress disorder: Symptoms for 3 days to 1 month
PTSD: Symptoms for >1 month

6. An 18-year-old woman has had fever, mild nonproductive cough…

Epstein-Barr virus infection

Patient with fever, malaise, exudative pharyngitis, splenomegaly and prominent


cervical lymphadenopathy, most consistent with infectious mononucleosis (EBV >
CMV)
Strep throat would not last for 5 weeks
Diphtheria –> Pseudomembranes often in an immigrant patient

7. A 23-year-old man is brought to the physician by his mother…

Brief psychotic disorder

Patient who experienced a stressful event 1 week ago who has experienced auditory
hallucinations of a somebody who was not killed, most consistent with brief
psychotic disorder
Key idea: Bereavement can lead to auditory hallucinations from the decesased, but
should not lead to such a decline in functioning as is seen in this vignette
Symptoms of psychosis: Delusions, hallucinations, disorganized speech,
disorganized/catatonic behavior, negative symptoms (flat affect, etc.)
Brief psychotic disorder: At least 1 positive symptom for less than 1 month (often
associated with stress)
Schizophreniform disorder: At least 2 psychosis symptoms for 1 month with decline
in functioning between 1 month to 6 months

https://step-prep.org/step-2-ck-ccssa-form-6-2/ Page 35 of 70
Step 2 CK (CCSSA) Form 7 – Step Prep 30/10/21, 12:51 AM

Schizophrenia: At least 2 psychosis symptoms for 1 month with decline in


functioning for at least 6 months

8. A 72-year-old man comes to the physician for a follow-up…

Psychogenic polydipsia

Older patient who has been drinking 12-15 glasses of water daily and has a history
of schizoaffective disorder found to have hyponatremia with low urine sodium and
low urine osmolality, most consistent with psychogenic polydipsia
Key idea: Despite UWorld claims, urine osmolality does NOT need to be below 100 to
diagnose psychogenic polydipsia
Key idea: Antipsychotics can commonly lead to dry mouth (as is seen in this
patient), making a patient feel thirsty

9. A 27-year-old primigravid woman at 39 weeks’ gestation is…

Cesarean delivery

Category 3 fetal heart tracings (require immediate delivery): (1) Recurrent late
decelerations + absent variability (2) Recurrent variable decelerations + absent
variability (3) Bradycardia + absent variability (4) Sinusoidal pattern
Cervical dilation < 10 cm and need for immediate delivery –> C-section
Cervical dilation of 10 cm and need for immediate delivery –> Operative vaginal
delivery (forceps delivery)
Key idea: Indications for C-section in the active phase (6-10 cm cervix) of labor (1)
Active phase arrest with no cervical change for 4 hours with adequate
contractions (>200 Montivideo units) or for 6 hours with inadequate contractions
(2) Category 3 fetal HR tracing

10. A 37-year-old woman comes to the physician because of a…

Bromocriptine therapy

Young woman with irregular menses, galactorrhea, bitemporal hemianopsia found

https://step-prep.org/step-2-ck-ccssa-form-6-2/ Page 36 of 70
Step 2 CK (CCSSA) Form 7 – Step Prep 30/10/21, 12:51 AM

to have elevated prolactin and a pituitary microadenoma, most consistent with a


prolactinoma
Key idea: Prolactin release from the anterior pituitary is inhibited by endogenous
dopamine release from the hypothalamus, so we give patients with a prolactinoma
dopamine agonists (bromocriptine, cabergoline) to inhibit prolactin release
Key idea: Patients with a macroprolactinoma (>10 mm) or symptomatic
microprolactinoma should be started on dopamine agonists and should get a
resection if the tumor is very large (>3 cm) or if the mass increases in size while on
a dopamine agonist

11. A 32-year-old woman with asthma comes to the physician…

Allergic bronchopulmonary aspergillosis

Allergic bronchopulmonary aspergillosis: Hypersensitivity response associated


with asthma and cystic fibrosis that can lead to bronchiectasis and eosinophils
Bronchiectasis = Chronic productive cough + linear atelectasis + thickened airways
and irregular cystic opacities
Key idea: Treatment is months of treatment with itraconazole + oral corticosteroids
Cystic fibrosis –> Also associated with bronchiectasis and ABPA, but patient would
have history of recurrent sinopulmonary infections, fat soluble vitamin deficiency,
etc.
Alpha-1 antitrypsin deficiency –> Lungs and liver problems with family history

12. A previously healthy 62-year-old man is brought to the…

Intravenous labetalol

Older male patient with tearing chest pain radiating to the pack, different upper
extremity blood pressures, aortic regurgitation murmur and a widened
mediastinum on CXR, all of which are consistent with aortic dissection
Key idea: Beta-blockers (such as labetalol) are particularly useful in setting of aortic
dissection because the force of blood being ejected from the LV and hitting the aorta
can further propagate the tear, with beta blockers leading to reduced contractility
and reduced force of blood ejection against the aorta

https://step-prep.org/step-2-ck-ccssa-form-6-2/ Page 37 of 70
Step 2 CK (CCSSA) Form 7 – Step Prep 30/10/21, 12:51 AM

13. A 47-year-old man comes to the physician because of a 9-month…

Colonoscopy

Key idea: Patients get their first screening colonoscopy at 50 years old, so in this
patient with bowel changes (constipation) and anemia (which cannot be fully
explained by hemorrhoids), the next best step would be a colonoscopy

14. A 37-year-old woman, gravida 3, para 3, comes for a routine…

Bartholin duct cyst

Location of mass most consistent with a bartholin duct cyst


Key idea: Always try to envision anatomic position when given anatomical terms
such as anterior, posterior, etc.

https://www.mayoclinic.org/diseases-conditions/bartholin-cyst/symptoms-causes/syc-20369976

15. An 8-year-old boy is brought to the physician 1 hour after the…

https://step-prep.org/step-2-ck-ccssa-form-6-2/ Page 38 of 70
Step 2 CK (CCSSA) Form 7 – Step Prep 30/10/21, 12:51 AM

C1 esterase inhibitor

Young patient with recurrent angioedema WITHOUT urticaria with a strong family
history, most consistent with hereditary angioedema which is caused by C1
inhibitor deficiency leading to impaired bradykinin breakdown, with increased
bradykinin –> angioedema (similar pathophysiology to ACE-inhibitor angioedema)
Key idea: Autosomal dominant inheritance pattern
Key idea: These patients also commonly present with colicky abdominal pain and GI
symptoms

16. An 18-year-old woman has had fever for 12 hours and obtundation…

Meningococcemia

Young woman who has been in a confined space with others (summer camp, college,
etc.) who presents with fever, altered mental status, hypotension and purpuric skin
lesions, most consistent with meningococcemia
Toxic shock syndrome –> Diffuse erythroderma
Lyme disease –> Erythema chronica migrans (“bulls-eye rash”)

17. Three days after open reduction and internal fixation of a right…

Duplex scan

Patient with recent surgery who presents with tightness/tenderness of the left calf,
most concerning for a DVT (which should be worked-up with a duplex ultrasound)

https://step-prep.org/step-2-ck-ccssa-form-6-2/ Page 39 of 70
Step 2 CK (CCSSA) Form 7 – Step Prep 30/10/21, 12:51 AM

https://en.wikipedia.org/wiki/Virchow%27s_triad

18. A 15-year-old girl is brought to the physician because of finger…

Systemic lupus erythematous

Young female patient with arthritis, malar rash, decreased hemoglobin and WBC
count, low complement, positive ANA and urinalysis consistent with a
glomerulonephritis, most consistent with systemic lupus erythematous
Mixed connective tissue disease: Autoimmune disease with variable features of
SLE, systemic sclerosis and polymyositis that is characterized by Raynaud
phenomenon, dactylitis, arthritis and inflammatory myopathy +/- other features

19. A 19-year-old African American man comes to the physician…

Digestive enzyme deficiency

Young African-American man with abdominal cramps/bloating with intermittent


watery diarrhea that has been worse since starting college (which likely came with
change in diet), most consistent with lactose intolerance
Key idea: Lactose intolerance is common in non-White populations (lactase
persistence more common in Northern European populations who historically eat a

https://step-prep.org/step-2-ck-ccssa-form-6-2/ Page 40 of 70
Step 2 CK (CCSSA) Form 7 – Step Prep 30/10/21, 12:51 AM

lot of dairy products)

20. A previously healthy 82-year-old woman comes to the physician…

Brisk rotatory nystagmus on left lateral gaze

Physiological nystagmus: Alternating smooth pursuit in one direction and saccadic


movement in the other direction (WOULD NOT BE ROTATORY)
All other answer changes represent normal changes in aging

21. A 16-year-old girl is brought to the emergency department after…

Aspirin

Patient who attempted an overdose who has a mixed anion-gap metabolic acidosis
(140 – 104 – 6 = 30 > 12) with a significant respiratory alkalosis, most consistent
with aspirin/salicylate poisoning

22. Four CAGE questions are used to screen for alcoholism...

Sensitivity: Increased // Specificity: Decreased

Number of yes answers needed for positive test moves from 2 –> 1 will lead to
decreased false negatives (increased sensitivity) and increased false positives
(decreased specificity)
Key idea: Low false negative –> high sensitivity (of patients with disease, how many
test positive), whereas low false positive –> high specificity (of patients without
disease, how many test negative)

23. Two days after admission to the hospital for detoxification…

Vitamin B1 (thiamine)

Patient with alcoholism presenting with ataxia (wide-based ataxic gait) and
ophthalmoplegia (abnormal eye movements), most consistent with Wernicke
encephalopathy (thiamine deficiency)

https://step-prep.org/step-2-ck-ccssa-form-6-2/ Page 41 of 70
Step 2 CK (CCSSA) Form 7 – Step Prep 30/10/21, 12:51 AM

Key idea: Alcoholism = Malnutrition (at least on NBME)


Wernicke encephalopathy: Confusion, ataxia, ophthalmoplegia

24. A previously healthy 14-year-old boy is brought to the physician…

Repeated microfracture at the tendon insertion

Adolescent boy with knee pain exacerbated by jumping who has


tenderness/swelling specifically over the tibial tubercle, most consistent with
Osgood-Schlatter disease (patellar tendon pulls on tibial tubercle repeatedly,
leading to inflammation and repeated microfractures)

https://kidshealth.org/en/parents/osgood.html

25. A 37-year-old woman with sickle cell disease comes to the…

Acute cholecystitis

Patient with sickle cell disease presenting with fever, RUQ pain, mild scleral icterus
and positive Murphy sign (specific for acute cholecystitis on NBME exams) with
an ultrasound showing cholelithiasis and pericholecystic fluid, most consistent with
acute cholecystitis
Key idea: Acute cholecystitis classically does NOT lead to jaundice or
hyperbilirubinemia, but this patient has another reason to have jaundice because
sickle cell disease –> chronic hemolysis –> unconjugated hyperbilirubinemia
Risk factors for cholesterol gallstones: Female, Fat, Forty, Fertile

https://step-prep.org/step-2-ck-ccssa-form-6-2/ Page 42 of 70
Step 2 CK (CCSSA) Form 7 – Step Prep 30/10/21, 12:51 AM

Risk factors for bilirubin gallstones –> Chronic hemolysis


Cholangitis –> Negative Murphy sign, signs of cholestasis (alkaline phosphatase >
ALT/AST), dilated common bile duct

26. Three hours after undergoing emergency repair of a leaking…

Cardiogenic shock

Patient with a major aortic injury who presents with decreased cardiac index,
hypotension, increased pulmonary capillary wedge pressure, increased CVP and
increased PA pressure with ECG changes most consistent with cardiogenic shock
Distributive shock (sepsis, anaphylaxis) –> Increased cardiac index, decreased PCWP,
decreased SVR
Hypovolemic shock –> Decreased cardiac index, decreased PCWP, increased SVR
CHF –> Pulmonary edema, lower extremity edema, elevated JVP, etc.

27. An otherwise healthy 70-year-old woman comes to the physician…

Paget disease of the breast

~50% of women with Paget disease of the breast will have a negative mammogram
and no masses on physical exam
Key idea: If patient had eczema, we would expect a longer history of eczema and
rashes in places other than just the nipple

28. An otherwise healthy 40-year-old woman comes to the physician…

Physiologic discharge

Physiologic discharge: Non-spontaneous, non-bloody and minimal discharge with


normal physical exam and mammogram
Pathologic discharge: Unilateral discharge, bloody/serous discharge and/or palpable
lump or skin changes

29. A 27-year-old man comes to the physician because of a 2-month…

https://step-prep.org/step-2-ck-ccssa-form-6-2/ Page 43 of 70
Step 2 CK (CCSSA) Form 7 – Step Prep 30/10/21, 12:51 AM

HIV antibody testing

Patient with disseminated molluscum contagiosum, which in an adult should prompt


concern for immunodeficiency, most likely HIV
Key idea: Increased severity/dissemination of various skin conditions (molluscum
contagiosum, psoriasis, tinea, seborrheic dermatitis, etc.) can be a sign of
immunodeficiency and should be worked up with HIV testing

30. A 72-year-old man comes to the physician because of progressive…

Aortic stenosis

Old patient with “parvus et tardus” (diminished carotid upstrokes) and a 4/6
systolic murmur heard throughout the precordium with radiation to the carotids,
most consistent with aortic stenosis
Key idea: Aortic stenosis can lead to heart failure because the LV will need to pump
against increased afterload and will eventually become weak
Key idea: Most common cause of aortic stenosis is aortic valve calcification that is
seen in elderly patients and developed over time

31. A 72-year-old man has decreased urine output 2 days after…

Blood: 1+ // Protein: 1+ // RBC: 0-5 // WBC: 0-5 // Casts: Pigmented granular //


Other Microscopic findings: Renal tubular epithelial cells

Elderly patient with gram-negative bacteremia receiving gentamicin


(aminoglycosides associated with acute tubular necrosis) who has developed
oliguria and an AKI, most consistent with acute tubular necrosis
Acute tubular necrosis leads to pigmented granular casts
Causes of acute tubular necrosis: Sepsis, Aminoglycosides, ischemia/hypotension
(spectrum of disease of pre-renal AKI), heavy metals, radiocontrast, ethylene glycol,
rhabdomyolysis

32. A 42-year-old man has had a pruritic rash on his back for 4 days…

https://step-prep.org/step-2-ck-ccssa-form-6-2/ Page 44 of 70
Step 2 CK (CCSSA) Form 7 – Step Prep 30/10/21, 12:51 AM

Cutaneous larva migrans

Patient who commonly has to maneuver on the ground while on his back who
presents with a pruritic serpiginous rash on his back and is found to have an
eosinophilia, most consistent with cutaneous larva migrans (caused by
Ancyclostoma or Necator hookworks)
Ascariasis –> Bowel obstruction, biliary obstruction, eosinophilia
Scabies –> Pruritic rash worse at night with burrows in web-spaces of fingers/toes

https://dermnetnz.org/topics/cutaneous-larva-migrans/

33. A 72-year-old woman comes to the physician because of an increase…

Exploratory laparotomy

Older woman with history of breast cancer found to have increasing abdominal girth,
ascites and a fixed non-tender adnexal mass most concerning for ovarian cancer
Key idea: Advanced ovarian cancer often spreads to the abdominal cavity,
so exploratory laparotomy with cancer resection and inspection is needed
for surgical staging

https://step-prep.org/step-2-ck-ccssa-form-6-2/ Page 45 of 70
Step 2 CK (CCSSA) Form 7 – Step Prep 30/10/21, 12:51 AM

Key idea: Image-guided biopsy is contraindicated, as it can predispose to abdominal


cavity seeding
Key idea: Ascites in a postmenopausal woman is always pathologic and is the origin
of the typical symptoms of early satiety, weight gain, etc. seen in advanced ovarian
cancer

34. Three days after hospitalization for treatment of severe muscle weakness…

Elevation of the head of the bed

Patient with severe muscle weakness who has developed fever, hypoxia, new
infiltrates in the middle/lower lobes and a polymicrobial infection via
bronchoscopy, most consistent with aspiration pneumonia

35. A 67-year old man comes to the physician because of easy fatigability…

Streptococcus pneumoniae

Older patient with multiple myeloma (monoclonal spike on SPEP and bone marrow
biopsy showing >50% plasma cells), which leads to impaired production of normal
immunoglobulins, leading to increased risk for sinopulmonary infections (due to
impaired IgA production) and encapsulated infections (due to impaired IgG –>
impaired opsonization –> increased risk of encapsulated infections such as Strep
pneumo, H. influenzae and Neisseria)

36. A 24-year-old primigravid woman at 38 weeks’ gestation is…

Intravenous penicillin G

Indications for intrapartum prophylaxis against GBS infection: (1) GBS-positive


rectovaginal culture during current pregnancy (2) GBS bacteriuria or GBS UTI in
current pregnancy (3) Unknown GBS status + <37 weeks gestation OR intrapartum
fever OR rupture of membranes for >18 hours
GBS with no allergy: IV penicillin
GBS with mild penicillin allergy (prior maculopapular rash without pruritis):

https://step-prep.org/step-2-ck-ccssa-form-6-2/ Page 46 of 70
Step 2 CK (CCSSA) Form 7 – Step Prep 30/10/21, 12:51 AM

Cefazolin
GBS with severe penicillin allergy (high risk of anaphylaxis, respiratory distress and
urticaria): Clindamycin/erythromycin

37. A 14-year-old boy who has sickle cell trait is brought to the emergency…

X-ray of the left hip

Young patient with sickle cell trait presenting with 1-day of atraumatic thigh and knee
pain with limited hip range of motion, concerning for avascular necrosis of the left
hip
Causes of avascular necrosis: CASTS Bent LEGS (Corticosteroids, Alcohol, Sickle
cell disease, Trauma, SLE, “the Bends”, LEgg-Calve-Perthes disease, Gaucher
disease, SCFE

38. A 77-year-old man is brought to the physician by family members…

Hypothyroidism

Older patient with dementia, decreased DTRs, non-pitting edema of the lower
extremities and slow movements, most consistent with hypothyroidism
Huntington disease –> Dementia, depression, chorea
Multi-infarct (vascular) dementia –> Step-wise dementia with focal neurologic
defects (weakness, sensation, etc.)
Pernicious anemia –> Atrophic gastritis + B12 deficiency (lower extremity weakness,
paresthesias, sensory disturbances, etc.)
Syphilis –> Lymphadenopathy, diffuse rash involving the palms and soles

39. A 42-year-old man is scheduled to undergo an MRI for evaluation…

Lorazepam therapy

Patient who developed symptoms of a panic attack while in an MRI (which involves
being placed into a tight space and can lead to severe claustrophobia) with no history
of previous illicit drug use or addiction, who could benefit from an acute-acting

https://step-prep.org/step-2-ck-ccssa-form-6-2/ Page 47 of 70
Step 2 CK (CCSSA) Form 7 – Step Prep 30/10/21, 12:51 AM

benzodiazepine for relaxation

40. Five days after undergoing an open splenectomy for immune…

X-rays of the chest

Patient with a recent surgery who develops dyspnea, decreased localized breath
sounds, and a leukocytosis most concerning for a hospital-acquired pneumonia,
which should be worked-up with chest x-ray
Pulmonary angiography (pulmonary embolism): Tachycardia, signs of DVT,
hemoptysis, etc.

41. A 62-year-old man comes to the physician because of intermittent…

Colonoscopy to the cecum

Key idea: Important to rule out multiple masses, as that could affect surgical
approach/management

42. A previously healthy 37-year-old woman comes to the physician…

Peristalsis: Decreased // Lower esophageal sphincter tone: increased

Previously healthy young woman with dysphagia to solids and liquids with upper
endoscopy showing retained secretions in the esophagus with barium swallow
showing “bird’s beak sign”, all of which is most consistent with achalasia
Key idea: Achalasia characterized by absence of esophageal peristalsis and
impaired relaxation of lower esophageal sphincter

43. Three days after undergoing total hip replacement, a 50-year-old…

Epidural hematoma

Patient who recently had an epidural catheter removed who has developed inability
to move his legs and urinary incontinence, most consistent with an epidural
hematoma –> spinal cord compression –> lower extremity weakness and overflow

https://step-prep.org/step-2-ck-ccssa-form-6-2/ Page 48 of 70
Step 2 CK (CCSSA) Form 7 – Step Prep 30/10/21, 12:51 AM

incontinence due to impaired bladder contraction

44. A 67-year-old man comes to the physician because of aching…

Femoropopliteal arteries

Femoropopliteal artery plaque –> Calf claudication


External iliac artery plaque –> Thigh and calf claudication
Aortoiliac plaque –> Gluteal, thigh and calf claudication; impotence

45. A 37-year-old man comes to the physician because of persistent…

Calcium disodium edetate

Young patient who renovates old houses who presents with abdominal pain, wrist
drop, sensory defects and a microcytic hypochromic anemia, most consistent with
lead poisoning
Symptoms for lead poisoning: LEAD (Lead lines on gingivae and metaphyses of long
bones, Encephalopathy and Erythrocyte basophilic stippling, Abdominal pain and
sideroblastic Anemia and wrist Drop (with other neuro symptoms)
Lead poisoning treatments: Dimercaprol, EDTA (calcium sodium edetate = calcium
sodium EDTA), succimer
Key idea: Potential buzzwords that should trigger thought of lead poisoning
include battery factory, homemade whiskey, renovating old house, etc.

46. An otherwise healthy 42-year-old woman comes to the physician…

Reassurance

Otherwise healthy woman with an atraumatic, asymptomatic red eye upon waking,
most consistent with subconjunctival hemorrhage
Key idea: Often results due to simple trauma from rubbing eyes vigorously and
violent coughing spells, but on NBME often apparent upon waking
Key idea: Managed by observation because it typically resolves within 24-48 hours

Exam section 4:

https://step-prep.org/step-2-ck-ccssa-form-6-2/ Page 49 of 70
Step 2 CK (CCSSA) Form 7 – Step Prep 30/10/21, 12:51 AM

1: An 82-year-old man with congestive heart failure (CHF) comes to the


physician…

Decreased renal blood flow

CHF patient with increased diuretic dosage –> Too much fluid is pulled off –>
Decreased effective circulating volume –> AKI (pre-renal, acute tubular necrosis)
Key idea: When taking care of a patient with decompensated heart failure, important
to balance correcting volume overload with risk of causing an AKI

2. A previously healthy 68-year-old man is brought to the emergency


department…

Acute myocardial infarction

Older man presenting with acute substernal chest pain radiating to his left arm
(classic for MI), diaphoresis and an EKG showing ST elevation in leads 1, V2-V6,
consistent with an anterior MI (left anterior descending involvement)

3. A 70-year-old woman is brought to the emergency department 3 hours…

Ventricular tachycardia

Older woman with known CAD who presents with acute chest pain, weakness and
dyspnea who is found to be hypotensive with an ECG with a rate of 150 (2 big
boxes b/w QRS complexes) and a wide QRS (>120 ms), most consistent with
ventricular tachycardia likely secondary to coronary ischemia

4. A 52-year-old woman with a 2-year history of type 2 diabetes mellitus comes…

Add lisinopril to the medication regimen

Key idea: ACE inhibitors or ARBs should be initiated at onset of microalbuminuria in


all diabetic patients
Key idea: Two most common causes of CKD include hypertension and diabetes, with
diabetes leading to nephropathy because it leads to preferential arteriosclerosis of

https://step-prep.org/step-2-ck-ccssa-form-6-2/ Page 50 of 70
Step 2 CK (CCSSA) Form 7 – Step Prep 30/10/21, 12:51 AM

the efferent arteriole > afferent arteriole, thus leading to high pressure in glomerulus,
resulting elevated GFR and damage to glomerulus over time (ACE inhibitor or ARB
can reduce this damage by dilating the efferent arteriole, leading to a decreased
GFR)
Key idea: Typical HbA1c goal for patients with diabetes is <7.0% (and therefore in
this patient no need to switch diabetes meds)

5. A 20-year-old man is brought to the physician by his parents because of…

Schizophrenia

Young male patient (perfect demographic for onset of schizophrenia) who presents
with 1 year of auditory hallucinations and negative symptoms, most consistent
with schizophrenia
TSH is normal, so patient would not have psychosis secondary to thyroid dysfunction
// Urine toxicology is normal so would not be substance-induced psychotic disorder
Brief psychotic disorder: 3 days to 1 month
Schizophreniform disorder: 1 month to 6 months
Schizophrenia: >6 months

6. A 32-year-old man is brought to the emergency department by a friend…

Dopamine

Young patient with recent medication addition to treat hallucinations (most likely an
antipsychotic), who has fever, encephalopathy, unstable vitals (tachycardia,
hypertension), elevated CK, and muscle rigidity, all of which are most consistent
with neuroleptic malignant syndrome (secondary to antipsychotics that are
dopamine antagonists)
Serotonin syndrome: 3 A’s (Autonomic dysregulation, Agitation and increased
Activity (hyperreflexia, hypertonia, tremor, etc.) in a patient taking any drug that
increases serotonin (SSRIs, SNRIs, TCAs, MAO inhibitors, triptans, MDMA,
dextromethorphan, meperidine, St. John’s wort, ondansetron, tramadol, linezolid, etc.)

7. A 2-month-old girl is brought for a routine health maintenance examination…

https://step-prep.org/step-2-ck-ccssa-form-6-2/ Page 51 of 70
Step 2 CK (CCSSA) Form 7 – Step Prep 30/10/21, 12:51 AM

Give all recommended immunizations for age

Key idea: Premature infants receive immunizations based on chronologic age (age
since birth), with key exception being that they do not receive hepatitis B vaccine
until they are at least 2 kg in weight
Key idea: Children with low-grade fever (<101 F) or mild illness (cold, runny nose,
otitis media, mild diarrhea) can still get vaccines

8. A previously healthy 10-year-old girl is brought to the physician because…

No pharmacotherapy is indicated at this time

Previously healthy child with 2 day history of a “slapped cheek” rash that has spread
to the reset of the body, low-grade fever and malaise, most consistent with
Parvovirus B19 which is not treated with medication
Key idea: Although Parvovirus most associated with “slapped cheek” rash, it also can
lead to reticular, erythematous rash on their extremities and trunk
Reminder: Parvovirus B19 associated with development of aplastic anemia

9. An asymptomatic 60-year-old man comes for a follow-up examination 2


months…

Observation

Patient who is asymptomatic with a hiatal hernia that can be managed with
observation
Key idea: Patients with a symptomatic hiatal hernia (GERD), should be treated with
PPIs or H2 blockers

10. A 22-year-old college student is brought to the student health clinic…

Olanzapine therapy

Young male patient (classic demographic for schizophrenia) with history of marijuana
use (which increases risk for development of schizophrenia) who presents with 1-
month of delusions (contaminated food), auditory hallucinations and impaired

https://step-prep.org/step-2-ck-ccssa-form-6-2/ Page 52 of 70
Step 2 CK (CCSSA) Form 7 – Step Prep 30/10/21, 12:51 AM

functioning (school performance deteriorated), most consistent with


schizophreniform disorder –> Treat with antipsychotic (of which olanzapine is the
only one listed)

11. A 32-year-old woman is brought to the emergency department because of…

Appendicitis

Young woman with 2-day history of fever, nausea/vomiting, RLQ abdominal pain with
rebound and a neutrophilic leukocytosis, most consistent with acute appendicitis
Negative ultrasound for adnexal mass –> Not adnexal torsion, corpus luteum cyst,
degenerating leiomyoma uteri, tubo-ovarian abscess, etc.

12. An unconscious 22-year-old man is brought to the emergency department…

Intravenous 0.9% saline

Patient engaging in intense physical activity presenting with signs of heat stroke
(fever, seizure/encephalopathy, dry hot skin) and rhabdomyolysis (myoglobinuria),
who should be treated with IV fluids to decrease the risk of myoglobin pigment
stones precipitating kidney damage
Key idea: The major tenant of rhabdomyolysis management is aggressive IV fluid
resuscitation because patients develop decreased effective circulating volume
(due to fluid shifts into damaged muscle) and because patients are at risk for acute
tubular necrosis secondary to myoglobinuria (with increased fluids helping to dilute
these pigments and decrease the risk of tubular obstruction)

13. A 57-year-old man comes to the physician because of generalized weakness..

Interferon-alfa

Patient with signs/symptoms of mixed cryoglobulinemia (weakness, arthralgias,


palpable purpura, positive cryoglobulins on labs) most likely secondary to hepatitis C
contracted during blood transfusion (positive Hep C viral RNA), and therefore the
patient should receive interferon-alpha therapy against the hepatitis C

https://step-prep.org/step-2-ck-ccssa-form-6-2/ Page 53 of 70
Step 2 CK (CCSSA) Form 7 – Step Prep 30/10/21, 12:51 AM

Key idea: This test question is a bit outdated, with hepatitis C now being treated with
combination of NS5A/B inhibitors (ledipasvir, velpatasvir, sofosbuvir, etc.) and
NS3/4A inhibitors (simeprevir, etc.)
Key idea: To confirm diagnosis of hepatitis C, antibody testing needs to be followed
up by viral RNA because hepatitis C will be spontaneously cleared by the body in 15-
25% (no chronic Hep C)

14. A previously healthy 27-year-old African American woman, gravida 3…

Abruptio placentae

Woman at 34 weeks’ gestation with abdominal pain and vaginal bleeding (which
should sound off alarm bells for placental abruption), most consistent with
placental abruption
Anterior placenta and abdominal pain are inconsistent with placenta previa
Preeclampsia can lead to abruptio placenta, and that may be the case here
Risk factors for placental abruption: Maternal hypertension/pre-
eclampsia/eclampsia, abdominal trauma, prior placental abruption, drug use (cocaine,
tobacco)
Management of placental abruption: Aggressive maternal fluid resuscitation with
crystalloids + delivery

15. A 16-year-old girl has had increasingly severe abdominal pain for 2 days…

Pelvic inflammatory disease

Adolescent girl who is sexually active with inconsistent contraceptive use who
presents with fever, pharyngeal erythema and tenderness in the RLQ with cervical
motion tenderness and a negative pregnancy test, most consistent with pelvic
inflammatory disease (with likely gonorrhea co-infection of the throat)
Cervical motion tenderness: Pelvic inflammatory disease or ectopic pregnancy
(negative pregnancy test rules it out)
Disseminated gonococcemia: Dermatitis, oligoarthritis, and enthesitis OR
monoarticular septic arthritis

https://step-prep.org/step-2-ck-ccssa-form-6-2/ Page 54 of 70
Step 2 CK (CCSSA) Form 7 – Step Prep 30/10/21, 12:51 AM

16. A 52-year-old woman comes to the physician because of a 2-week history…

Pleural metastases

Middle aged woman with history of breast cancer presenting with subacute dyspnea
on exertion with dullness to percussion over the lower half of the right lung
(consistent with pleural effusion) with no other sides of volume overload, most
consistent with pleural effusion secondary to metastatic carcinoma
Pericardial tamponade: Hypotension, jugular venous distention, distant heart
sounds, pulsus paradoxus
RLL pneumonia: Fever, cough, etc.

17. A 56-year-old woman with short-bowel syndrome caused by mesenteric…

Zinc

Signs of zinc deficiency: Alopecia, dermatitis, scaly skin around mouth/eyes,


abnormal taste, impaired wound healing
Signs of copper deficiency: Hematologic abnormalities (microcytic anemia,
leukopenia) and myeloneuropathy
Iron deficiency: Microcytic anemia

18. A previously healthy 18-year-old woman comes to the physician because…

Increased serum lactate dehydrogenase activity

Young previously healthy woman with a large (>1 cm), firm supraclavicular lymph
node and splenomegaly, most consistent with Hodgkin lymphoma (which is
associated with an increased LDH)
Key idea: Enlarged supraclavicular lymph node is never normal!
Key idea: Hodgkin lymphoma also commonly associated with pruritis (particularly
on the NBME exam)

19. A 6-week-old boy is brought to the physician for his first well-child
examination…

https://step-prep.org/step-2-ck-ccssa-form-6-2/ Page 55 of 70
Step 2 CK (CCSSA) Form 7 – Step Prep 30/10/21, 12:51 AM

Fibrosis of the sternomastoid muscle

6 week old newborn who had a difficult delivery via forceps who now has his head
rotated to the left with contralateral chin deviation with a nontender mass in the
right side of the neck, most consistent with torticollis
Torticollis: Neck mass + ipsilateral head tilt + contralateral chin deviation

https://www.correctivechiropractic.com/blog/can-chiropractic-help-torticollis-infants-children/

20. A 3-year-old girl is brought to the emergency department after her father…

Organophosphate

Young child who was unsupervised with chemicals and presents with cyanosis,
diaphoresis, miosis, rhinorrhea, drooling, signs of bronchoconstriction, and increased
bowel sounds, all of which is associated with organophosphate poisoning –>
Decreased acetylcholinesterase activity –> Increased acetylcholine –> Increased
parasympathetic functions, increased sweating, muscle paralysis (due to
repetitive stimulation)
Key idea: Treated with atropine (muscarinic receptor antagonist) and pralidoxime
(regenerates acetylcholinesterase)

https://step-prep.org/step-2-ck-ccssa-form-6-2/ Page 56 of 70
Step 2 CK (CCSSA) Form 7 – Step Prep 30/10/21, 12:51 AM

Lead poisoning would not present acutely


Ethylene glycol –> Anion-gap metabolic acidosis and kidney injury (classically renal
stone)
Carbon monoxide poisoning –> Cherry-red skin, head, altered mental status

21. A 32-year-old woman, gravida 3, para 1, aborta 1, is brought to the


emergency…

Magnesium sulfate

Woman with history of preterm labor who presents with preterm labor (contractions
AND cervical changes) at 28 weeks’ gestation and therefore should receive (1)
Betamethasone (promote lung development) (2) Magnesium sulfate
(neuroprotective for baby, tocolytic effects (3) Tocolytics (4) IV Penicillin if GBS
positive or unknown
Delivery at <32 weeks: See above
Delivery at 32-34: (1) Betamethasone (promote lung development) (2) Tocolytics (3)
IV penicillin if GBS positive or unknown
Delivery at 34-37: (1) Betamethasone (promote lung development) (2) IV penicillin if
GBS positive or unknown

22. A male newborn has bilateral clubfoot deformity. He was born at term
following…

Spinal dysraphism

Male newborn who has lower limb paralysis and numbness with a full bladder, most
consistent with a spinal cord injury affecting the lower extremities
Spinal dysraphism = Type of neural tube defect
Cerebral palsy: Delayed motor milestones, comorbid seizures/intellectual disability,
abnormal muscle tone
Muscular dystrophy: Waddling gait, hip weakness, Gower sign in a toddler

23. A 42-year-old woman comes to the physician because of a 3-month history


of…

https://step-prep.org/step-2-ck-ccssa-form-6-2/ Page 57 of 70
Step 2 CK (CCSSA) Form 7 – Step Prep 30/10/21, 12:51 AM

Oral propylthiouracil therapy

Young woman with subacute palpitations, anxiety, weight loss, heat intolerance,
diarrhea, lid lag and exophthalmos with a large, nontender thyroid gland with
diffuse increased uptake and low TSH with increased T4 levels, most consistent
with Graves’ disease
Graves disease and other forms of hyperthyroidism should be treated with
propylthiouracil or methimazole
Key idea: All causes of hyperthyroidism can lead to lid lag and lid
retraction secondary to sympathetic activation of the superior tarsal muscle, but
Graves specifically leads to exophthalmos and impaired extra-ocular motion due to
glycosaminoglycan deposition behind the eyes

https://www.aafp.org/afp/2016/0301/p363.html

https://step-prep.org/step-2-ck-ccssa-form-6-2/ Page 58 of 70
Step 2 CK (CCSSA) Form 7 – Step Prep 30/10/21, 12:51 AM

24. A 42-year-old woman is brought to the emergency department because of…

Optic neuritis

Young woman with history of neurologic symptoms disseminated in time and


space who presents with left eye pain and vision changes who has a central
scotoma, an afferent pupillary defect and pallor of the left optic disc most
consistent with optic neuritis in the setting of multiple sclerosis
Macular degeneration: Older patient with loss of central vision and straight lines
appearing curved
Retinal detachment: Patient with history of “flashes and floaters” who presents with
monocular vision loss like a “curtain drawn down”

25. A 47-year-old man comes to the emergency department because of a 3-day…

Hypovolemia

Patient with a 3-day history of profuse nausea/vomiting after significant alcohol


consumption who now presents with abdominal pain, hypotension, tachycardia and
cool/clammy skin, most consistent with hypovolemic shock
Cardiogenic shock: Signs of volume overload (jugular venous distention, lower
extremity edema, etc.)
Septic shock: Could be seen if patient had sign of perforation (guarding, rebound)
and patient would likely have warm and dry skin (because cardiac output is
normal/increased and peripheral arterioles are dilated)

26. An otherwise healthy 37-year-old man comes to the physician because of…

Spondylolisthesis

Spondylolisthesis: Palpable step-off, pain with spinal extension, with lateral spinal x-
ray showing forward slippage of one of the vertebral bodies
Key idea: Due to pars intercularis defects that causes forward slip of vertebral body,
leading to nerve impingement /// Often seen in adolescent boys
Compression fracture: Patient with risk factors for osteoporosis with tenderness

https://step-prep.org/step-2-ck-ccssa-form-6-2/ Page 59 of 70
Step 2 CK (CCSSA) Form 7 – Step Prep 30/10/21, 12:51 AM

upon palpation of the spine and pain with sitting/standing

http://www.learningradiology.com/archives06/COW%20204-
Spondolytic%20Spondylolisthesis/spondylocorrect.htm

27. A previously healthy 21-year-old woman comes to the physician 1 day…

Viral pleurisy

Previously healthy young patient presenting with severe pleuritic chest pain,
shortness-of-breath, mild fever and a faint erythematous rash, consistent with a viral
syndrome leading to viral pleurisy and a viral exanthem
Key idea: Pleural friction rub = Pleural inflammation = Pleurisy (similar to how
pericardial friction rub = pericarditis)

https://step-prep.org/step-2-ck-ccssa-form-6-2/ Page 60 of 70
Step 2 CK (CCSSA) Form 7 – Step Prep 30/10/21, 12:51 AM

28. A 17-year-old boy comes to the physician because of a 4-day history of


severe…

Pill-induced esophagitis

Patient who has been taking ibuprofen (NSAIDs) who presents with painful
swallowing with no other findings, most concerning for pill-induced esophagitis
Causes of pill-induced esophagitis: Tetracyclines, Bisphosphonates, potassium
chloride, NSAIDs, Iron
Zenker diverticulum: Older patient with halitosis, regurgitation of food +/- palpable
neck mass

29. A 27-year-old nulligravid woman comes for a routine health maintenance…

Colposcopy

Negative pap smear: Repeat screen in 3 years (or 5 years with HPV co-testing in
women 30-65 years old)
Atypical squamous cells of undetermined significance (ASCUS): Either perform
HPV testing or repeat pap within 6 months
Abnormal pap smear (LSIL or worse): Confirmatory colposcopy –> Cryotherapy or
loop electrosurgical excision procedure (LEEP) if patient has local ectocervical
disease vs. Cone biopsy if patient has local endocervical disease

30. A 27-year-old man is brought to the emergency department by parademics


30…

Transfusion of group O, Rh-negative packed red blood cells

Key idea: Patient has active bleed into chest (right hemopneumothorax with
negative FAST) that is not responding to fluids (remains tachycardic and
hypotensive after 3L crystalloid), and therefore should be managed with blood
product resuscitation + surgical intervention
Key idea: Group O, Rh-negative blood can be given to patients of all ABO and Rh
blood types

https://step-prep.org/step-2-ck-ccssa-form-6-2/ Page 61 of 70
Step 2 CK (CCSSA) Form 7 – Step Prep 30/10/21, 12:51 AM

31. A 77-year-old man is brought to the emergency department 15 minutes


after…

Destruction and dilation of distal airways

Older patient with a lifelong history of recurrent episodes of pneumonia and


chronic productive cough (which is consistent with bronchiectasis) who presents
with self-limited large-volume hemoptysis and is found to have thin-walled cystic
spaces in the RLL with air-fluid levels on chest x-ray, most consistent with
bronchiectasis due to airway destruction/dilation secondary to repeated
pneumonia
Key idea: Chronic productive cough = Bronchiectasis (recurrent infections, cystic
fibrosis, etc.) or chronic bronchitis (significant smoking history)

https://en.wikipedia.org/wiki/Bronchiectasis

https://step-prep.org/step-2-ck-ccssa-form-6-2/ Page 62 of 70
Step 2 CK (CCSSA) Form 7 – Step Prep 30/10/21, 12:51 AM

32. A 10-year-old girl is admitted to the hospital because of fever and joint pain…

Mitral valve incompetence

Child with strep throat (throat culture and increased ASO titer) who presents with
joint pain and a new 3/6 holosystolic murmur at the midclavicular line, most
consistent with mitral valve regurgitation caused by rheumatic heart disease
Rheumatic heart disease: JONES criteria (Joint pain, O for heart (endocarditis,
myocarditis, pericarditis), subcutaneous Nodules, Erythema marginatum, and
Sydenham chorea
Systolic murmurs (left-sided): Aortic stenosis vs mitral regurgitation

33. A 4-year-old boy develops chickenpox 8 hours after visiting her newborn…

No intervention is necessary

Key idea: All mothers have previous exposure to chickenpox and should theoretically
have protective IgG that they can pass to their newborns
Key idea: Only time newborn should receive prophylactic VZV immunoglobulin is if
mother developed chickenpox rash <5 days before delivery, mother developed
chickenpox rash <2 days after delivery or if baby were born to a non-immune
mother and was exposed to somebody with chickenpox

34. An 87-year-old man comes to the physician because of a 1-year history of…

Finasteride

Older man with nocturia, hesitancy, and uncontrollable leaking after the end of
urination who is found to have an enlarged prostate on exam, most consistent with
benign prostatic hyperplasia
2 potential treatments are (1) 5-alpha reductase inhibitors (finasteride, dutasteride):
Lead to reduced conversion of testosterone –> DHT –> Less BPH over the course of
months (2) Alpha-1 selective inhibitors (-zosin and tamsulosin): Relaxation of the
bladder internal sphincter –> less urethral resistance when voiding
Key idea: -zosin drugs (NOT TAMSULOSIN) also have blood pressure effects

https://step-prep.org/step-2-ck-ccssa-form-6-2/ Page 63 of 70
Step 2 CK (CCSSA) Form 7 – Step Prep 30/10/21, 12:51 AM

(because alpha-1 receptors lead to arteriolar constriction) and commonly lead to


orthostatic hypotension and dizziness, so in this patient with orthostatic
hypotension due to autonomic dysfunction, we would opt for tamsulosin or a 5-
alpha reductase inhibitor

35. A 67-year-old man has had an ulcer on the anterior surface of the leg just
above…

Stasis dermatitis with ulcer

Elderly patient with previous mitral valve dysfunction and history of CHF who
presents with bilateral lower extremity edema and an ulcer on the lower medial
extremity, most consistent with a venous stasis ulcer
3 main types of lower extremity ulcers include (1) Neuropathic ulcers: Seen in
diabetic patient, occur on the sole of the foot especially where lots of pressure is
placed while walking (such as proximal to big toe) (2) Arterial ulcers: Patient with
signs of peripheral vascular disease (claudication, shiny hairless legs) who has ulcer
with necrosis often at tips of the toes (3) Venous ulcers: Often seen in patient with
chronic lower extremity edema and leads to ulceration on the medial aspect of the
leg

36. A study is proposed to assess the effectiveness of a new vaccine for the
prevention…

Coercion of a vulnerable population

Prisoners, homeless patients, elderly patients, immigrants, etc. are vulnerable


populations

37. A previously healthy 67-year-old man has had an aching burning sensation
in…

Bronchogenic carcinoma

Older patient with clubbing and painful arthropathy of large joints in the lower

https://step-prep.org/step-2-ck-ccssa-form-6-2/ Page 64 of 70
Step 2 CK (CCSSA) Form 7 – Step Prep 30/10/21, 12:51 AM

extremity relieved by elevation, most consistent with hypertrophic pulmonary


osteoarthropathy secondary to lung cancer
Key idea: Next best step would be chest x-ray to look for lung cancer

38. A 65-year-old man comes to the emergency department 30 minutes after an


episode…

Internal carotid artery

Elderly man with vasculopathic risk factors (elevated cholesterol, type 2 diabetes)
who presents with transient blindness of the left eye (suggestive of amaurosis
fugax due to embolic phenomenon ipsilaterally on the left side) and an episode of
aphasia (suggestive of embolic phenomenon affecting the left side of the brain
where Broca’s area is located) also found to have a Hollenhorst plaque in the left eye,
all of which is consistent with carotid artery stenosis of the left internal carotid
artery throwing off small plaques
Key idea: Internal carotid artery will become the anterior cerebral artery and the
middle cerebral artery
Key idea: Amaurosis fugax is highly associated with carotid artery stenosis and is
due to occlusion of the ophthalmic artery, which is the first branch off the
INTERNAL carotid artery

39. A 62-year-old woman is brought to the emergency department because of a


4-day…

Radiation therapy

Older woman with known history of metastatic breast cancer (lymph node positive)
who presents with progressive back pain with lower extremity UMN weakness
(positive Babinski, brink reflexes) and lower extremity numbness/tingling,
concerning for metastases to the vertebral bodies leading to spinal cord
compression
Key idea: Bone metastatic disease should be managed with radiation therapy
https://www.ncbi.nlm.nih.gov/pmc/articles/PMC5421962/

https://step-prep.org/step-2-ck-ccssa-form-6-2/ Page 65 of 70
Step 2 CK (CCSSA) Form 7 – Step Prep 30/10/21, 12:51 AM

40. A 51-year-old woman had a 15-minute episode of acute right-sided chest


pain…

Observation

Middle-aged woman who had a right subclavian catheter placed (which is common
NBME risk factor for pneumothorax) and then developed right-sided chest pain and
dyspnea with normal O2 saturation and a small (10%) apical pneumothorax who
should be managed with observation
Key idea: If patient had a tension pneumothorax (medical emergency requiring
immediate needle decompression or chest tube placement), they would have
hemodynamic instability and tracheal deviation away from the affected side
Key idea: Patients with a small, largely asymptomatic pneumothorax can be
conservatively managed with observation and repeat chest x-ray hours later
https://www.mrinz.ac.nz/2020/01/30/psp-nejm/

41. A 62-year-old man comes to the physician because of a 12-hour history…

Vancomycin

Patient with immunosuppression (chemotherapy) and repeated insertion of a


catheter (notorious for causing nosocomial Staph and Strep infections) who
presents with fever and discharge from catheter site found to have an infection
with gram-positive cocci in clusters (= Staph aureus or Staph epidermidis) who
should be initially treated with Vancomycin to cover MRSA until sensitivities return
Key idea: Rates of MRSA are higher in patients who develop nosocomial Staph
infections compared to community Staph infections
The only cephalosporin that covers MRSA is ceftaroline (5th generation
cephalosporin)

42. A 25-year-old woman comes to the physician because of tremulousness and


fatigue…

Surreptitious administration of thyroxine

https://step-prep.org/step-2-ck-ccssa-form-6-2/ Page 66 of 70
Step 2 CK (CCSSA) Form 7 – Step Prep 30/10/21, 12:51 AM

These patients have a small thyroid because chronically low TSH leads to low levels
of thyroid stimulation and have low iodine uptake into the thyroid because the
thyroid is not actively producing thyroid hormone (thyroid hormone is coming into
body exogenously)
Key idea: We would expect this patient to have low TSH because of negative
feedback of exogenous T3/T4 upon anterior pituitary
Ophthalmic examination is normal = Not Graves disease

https://www.aafp.org/afp/2016/0301/p363.html

43. A 55-year-old man comes to the physician because of fever, neck pain, and…

Viral infection

https://step-prep.org/step-2-ck-ccssa-form-6-2/ Page 67 of 70
Step 2 CK (CCSSA) Form 7 – Step Prep 30/10/21, 12:51 AM

Patient with a recent viral infection who developed signs of hyperthyroidism (weight
loss) and an enlarged, tender thyroid gland, most consistent with DeQuervain
thyroiditis
Tender thyroid = DeQuervain thyroidits (subacute granulomatous thyroiditis) –>
Initially leads to hyperthyroidism due to release of preformed thyroid hormone (low
iodine uptake into thyroid gland) followed by hypothyroidism due to inflammation of
the thyroid gland

44. A 7-month-old boy is brought to the physician because of a 4-day history of…

Wiskott-Aldrich syndrome

Infant with recurring infections, eczema and thrombocytopenia, most consistent


with Wiskott-Aldrich syndrome
Adenosine deaminase deficiency = Severe combined immunodeficiency –> Failure to
thrive, recurrent infections with all types of organisms
Ataxia telangiectasia –> Cerebellar defects (ataxia), IgA deficiency (anaphylactic
blood transfusion reactions, sinopulmonary infections), spider angiomas
DiGeorge syndrome –> CATCH-22 (Cardiac defects, Abnormal faces, Thymic
aplasia, Cleft lip/palate, Hypocalcemia (due to lack of parathyroid glands)
Hyper-IgE –> Cold staph abscesses (contrast with abscesses without pus seen in
leukocyte adhesion defect), coarse faces, retained primary teeth, easy bone
fractures
Infant HIV –> Chronic diarrhea + failure to thrive + opportunistic infections (thrush,
etc.)
X-linked agammaglobulinemia –> Patient with history of uncles who die of infection
who develops recurrent bacterial (due to lack of IgG) and mucosal (due to lack of IgA)
infections found to have decreased levels of all immunoglobulins

45. A 72-year-old woman has hyponatremia 3 days after admission to the


hospital…

Inappropriate ADH (vasopressin) secretion

Elderly patient with cerebral disease who has hypo-osmolar hyponatremic with

https://step-prep.org/step-2-ck-ccssa-form-6-2/ Page 68 of 70
Step 2 CK (CCSSA) Form 7 – Step Prep 30/10/21, 12:51 AM

increased urine sodium (>40) and urine osmolarity >100, consistent with SIADH
Key idea: Causes of SIADH include ectopic ADH (small cell lung cancer), pulmonary
disease, CNS disorders or head trauma (such as infarction), and drugs
(cyclophosphamide)
Dehydration –> Low urine sodium (<40) because the body is trying to retain as much
sodium and water as possible
Water intoxication –> Low urine osmolality (often <100) because so much water is
being consumed and excreted that the urine is very dilute

46. A health status survey compares the clinical outcomes of patients treated…

The results are not adjusted for comorbidities

Process of elimination: Study period of 1 year does not seem short, results are
significant (p < 0.05), study is likely well-powered with sample size >500 and the
focus of the study is clinical (physical functioning, mobility)

We are not affiliated with the NBME, USMLE or AAMC.

The answer explanations may not be reproduced or distributed, in whole or in part, without written permission of

Step Prep.

Share this:

! Twitter " Facebook

Related

Step 2 CK (CCSSA) Form 6 NBME Family Med Form 1 Step 2 CK (CCSSA) Form 8
March 8, 2020 March 8, 2020 March 8, 2020
In "Step 2 CK (CCSSA) Form In "Family Medicine Forms" In "Step 2 CK (CCSSA) Form
6" 8"

ST E P 2 C K ( C C SSA ) FO R M 7

https://step-prep.org/step-2-ck-ccssa-form-6-2/ Page 69 of 70
Step 2 CK (CCSSA) Form 7 – Step Prep 30/10/21, 12:51 AM

https://step-prep.org/step-2-ck-ccssa-form-6-2/ Page 70 of 70

You might also like